MBE Practice Questions Hard/Answered Incorrectly

Réussis tes devoirs et examens dès maintenant avec Quizwiz!

Police went to a warehouse in response to a report of a gunshot. There they found the defendant standing over the victim's body. They immediately arrested the defendant and gave him Miranda warnings. The defendant said nothing other than that he wanted an attorney. The defendant eventually was charged with murder and brought to trial. At trial, the defendant testified in his own defense. He claimed that he was in the warehouse alone when he was attacked by the victim, and that he shot the victim in self-defense. The prosecution, on cross-examination, asked the defendant why he did not tell the police when they arrested him that he had shot the victim in self-defense. The defendant's attorney objected to this question, but he was overruled. The defendant was unable to give a satisfactory answer, and the prosecution suggested that he was lying. The defendant was convicted. Does the defendant have grounds to appeal his conviction on the basis of the prosecutor's cross-examination?

Miranda warnings carry implicit assurance that silence carries no penalty!!

A wealthy married couple living in State A mutually agreed that the marriage was not working out. The wife subsequently moved to State B. Approximately two years after moving to State B, the wife filed a divorce action in the United States District Court for the District of State A. As stated in the complaint filed in the matter, the marital estate that will be divided in the divorce is worth over $5 million. The husband filed a motion to dismiss for lack of subject matter jurisdiction. Should the motion to dismiss be granted?

Federal courts will NEVER take divorce, alimony, or child custody cases!

A defendant is on trial for a federal offense. The government has subpoenaed crucial documents from the defendant's bookkeeper that will clearly incriminate the defendant. May the defendant invoke her Fifth Amendment privilege against self-incrimination to prevent the documents from being admitted? Press Enter or Space to submit the answer

Fifth Amend privilege applies only to compelled testimony, not he production of documents that might incriminate. Physical evidence not protected, testimonial evidence is.

A defendant was charged with the murder of a victim. During the course of the criminal trial, a witness testified on behalf of the defense that, at the time the murder took place, he saw someone who looked like the defendant dancing at a local nightclub. The defendant is eventually acquitted of the charge. Following the acquittal, the appropriate survivors of the victim bring a wrongful death action against the defendant. As part of her defense, the defendant wishes to introduce the testimony given at the criminal trial by the witness, who the defendant shows is now incarcerated in a prison in another state. Is the testimony of the witness admissible?

Inadmissible here. For prior testimony to come in, the parties at issue must have had a sufficient opportunity and motive to cross examine the witness after giving that testimony. Here, different parties than the criminal case, no opportunity to develop testimony on cross.

Statutory Right of Redemption

Right of property owner to redeem for the foreclosure price after the foreclosure sale, not before.

Equitable right of redemption:

Right of property owner to redeem the land prior to foreclosure sale by paying off the amount owed to the creditor. CANNOT WAIVE THIS RIGHT. If there is an acceleration clause, must pay in full at time of default.

Part Performance Exception to SOF

The doctrine of part performance generally requires two of the following: possession, improvements, and full or partial payment of the purchase price.

Rule Regarding Subdivisions and Notice

When a developer subdivides land into several parcels and some of the deeds contain negative covenants but some do not, negative equitable servitudes binding all the parcels in the subdivision may be implied if: (i) there exists a common scheme for development, and (ii) the grantee has notice (inquiry, record, or actual) of the covenant.

A young man borrowed his mother's car to go on a job interview. He was running late and made a left turn directly in front of a taxicab without looking. The cab driver was busy talking on his cell phone and did not see the man until it was too late, so the two cars collided. Had the cab driver been paying proper attention, he could have avoided the accident. The cab's passenger was injured in the accident. The cab's passenger asserted a claim against the young man's mother for personal injuries. The jurisdiction retains the common law rule pertaining to the liability of car owners for the conduct of drivers. Will the passenger recover?

Common law rule is that an owner of a car is not liable for torts committed by another driver of that car!

A newly enacted federal law requires states to adopt a law banning texting while driving on interstate highways. Under the law, any state that does not enact such a law within three years will be denied 10% of the state's allotment of federal highway construction funding. What is the best argument that can be made in support of the constitutionality of this federal statute?

Congress can use its tax and spending power, as long as it isn't unduly coercive, to regulate states by imposing the grant of money on some sort of condition. Condition here was the adoption of law to ban texting and drivin.

A motorist was seriously injured when the "self-driving" car she was in collided with a stalled vehicle in its path. The car met all government safety standards for vehicles with driver-assisted technology but its sensors failed to detect the stopped vehicle in time. While the car was marketed as "self-driving," the car's instruction manual warned that operators must be in the driver's seat and ready at any time to take over control of the vehicle. While the motorist was in the driver's seat, she was distracted by a text on her phone and did not take over control of the car before the accident occurred. The motorist sued the car manufacturer in an action based on strict products liability. The car manufacturer moved for summary judgment and provided unrebutted evidence of the car's compliance with the government safety standards and the driver's failure to take control of the vehicle. How should the court respond to the motion?

In suing for strict liability, injury must not be a foreseeable misuse of the product to be a defense. This is a question of fact for the jury. Here, could easily be that texting while operating a self-driving car is a foreseeable misuse and the defendant would still be strictly liable.

A masked magician contracted with a casino to perform magic tricks six nights a week on the casino floor for $2,000 per week. The contract did not contain an assignment provision. One weekend, the magician came down with flu, so he asked his apprentice to perform in his place. The two signed a written assignment agreement that the apprentice would fully take the magician's place under the terms of the contract for the next week. Excited about the opportunity, the apprentice told the casino's manager of the deal, but no one else knew. During that week, the apprentice wore the magician's costume and mask and performed all of the magician's tricks. No one in the casino noticed the difference. When the magician regained his health, he and the apprentice had a falling out and the magician refused to pay the apprentice. The apprentice went to the casino with the assignment agreement and demanded his pay for the prior week's performances. The magician claimed that he was still the one entitled to payment under the contract. Who should the casino pay for the prior week's performances?

ASsignment here is valid because the apprentice told the manager and he accepted. Usually, assignment of special skills contract not valid but is here because of acceptance!

A buyer for a chain of shoe stores ordered 1,000 pairs of shoes from a shoe manufacturer. The shoes cost $50 per pair, so the total contract price was $50,000. It happened that the manufacturer owed $50,000 to a trucking company. The manufacturer assigned, in writing, "all proceeds from the contract with the buyer" to the trucking company. The manufacturer notified the buyer that he had assigned the proceeds of the contract to the trucking company and then shipped the 1,000 pairs of shoes to the buyer. Upon receipt of the shoes, the buyer discovered that 10% of the shoes were defective. He sent a check for 90% of the contract price ($45,000) to the manufacturer, who deposited the check. Shortly thereafter, the manufacturer closed down its business and disappeared without a trace. The trucking company, meanwhile, demanded payment from the buyer, to no avail. If the trucking company sues the buyer for the $45,000 that the buyer paid on the contract, will the trucking company prevail?

Buyer was given notice of assignment. At that point, payment to manufacturer (assignor) was done at their own peril and they still owed the assignee the balance that was shifted due to the assignment.

A tenant invited a friend over for dinner. On his arrival, the friend stepped on a split board on the front steps and the board broke, causing him to lose his balance and break his ankle. If the friend sues the tenant for his injuries and does not prevail in a jurisdiction that applies the traditional rules for landowners and possessors of land, what is the most likely explanation? A. In the lease, the landlord had undertaken the duty to discover and repair dangerous conditions on the premises. B. The friend arrived an hour earlier than his invitation specified. C. The friend should have noticed the dangerous condition himself. D. The tenant had stayed beyond the lease term and she no longer had the legal right to occupy the premises.

C. The friend should have noticed the dangerous condition himself. If the friend does not prevail, it will be because he should have noticed the dangerous condition himself. In jurisdictions applying the traditional rules for landowners and possessors of land, the nature of the duty owed by an owner or occupier of land to those on the premises for dangerous conditions on the land depends on the legal status of the plaintiff in regard to the property, i.e., trespasser, licensee, or invitee. A licensee is one who enters on the land with permission for his own purpose or business and includes social guests. The owner or occupier owes a licensee a duty to warn of or make safe a dangerous condition known to the owner or occupier that creates an unreasonable risk of harm to the licensee and that the licensee is unlikely to discover. The owner or occupier does not have a duty to inspect for defects or to repair known defects. Here, the friend had been invited for dinner, making him a licensee. The facts do not indicate whether the tenant knew of the split board and neglected to alert the friend or simply was not aware of it, but the duty to warn does not extend to dangerous conditions that the licensee should reasonably have discovered. Hence, (C) presents the best basis for the friend not prevailing. (A) is incorrect because that fact would not make a difference to the tenant's liability. The tenant remains liable to the friend for dangerous conditions on the premises as the occupier of the land, regardless of the landlord's obligation to inspect and repair. (B) is incorrect because the friend still qualifies as a licensee even though he arrived sooner than his invitation specified. It is true that a person may lose invitee status and become a licensee by being on the premises at a time outside the scope of his invitation. However, there is no similar principle applicable here. The fact that the friend arrived an hour early does not make him a trespasser rather than a licensee. Hence, the fact in (B) would not affect the tenant's liability. (D) is incorrect because it also is irrelevant. Even if the tenant had no legal right to occupy the land, she still would be the possessor of the land as to the friend, and she owed the friend the duties owed to a licensee.

During the defendant's trial for battery, a witness testified that he did not see the altercation but that he heard an eyewitness shout, "A fight! The man in the red shirt just punched the man in the yellow shirt in the face!" Previous evidence has established that the defendant was wearing a red shirt. The defendant offers to call a witness who will testify that he spoke with the eyewitness the day after the altercation and that the eyewitness said, "The man in the yellow shirt was punching the man in the red shirt." The eyewitness cannot be found to testify. Should this evidence be admitted?

Can come in. Do not need a hearsay declarant to be present to respond to their statement to impeach what they said. Here, witness testified as to hearsay declarant's statement, and the subsequent witness should be allowed to testify about his prior inconsistent statement even without the ability for him to explain or deny it.

A law student was sued by a student loan provider in federal court for failing to pay back her student loans. At the close of a federal civil trial, the court held in favor of the student loan provider for the full amount plus post-verdict interest at 8%. When the court clerk typed up the final order to send to all parties, she accidentally typed in 9% as the post-verdict interest rate. The law student did not notice the error until 14 months after the judgment was entered. She immediately filed a motion for relief from judgment based on a clerical mistake to correct the order to reflect the accurate interest rate. Will the court likely grant this motion? Press Enter or Space to submit the answer

Can make a motion to correct a clerical mistake at any time, does not matter how long after mistake was made!

A proprietor of a business in a small town who had a very antagonistic relationship with the mayor was denied a renewal of his business license. At a village council meeting that was closed to the public, the proprietor engaged in a heated argument with the mayor and demanded to know why his business license had not been renewed. The mayor truthfully declared that it was due to reports that the business was conducting illegal gambling on the premises. The mayor had received such reports but had not yet had them investigated. Council members at the meeting later voted to overturn the mayor's decision and renew the license. After a subsequent investigation, the reports were determined to be false. In a defamation action by the proprietor against the mayor, will the proprietor prevail?

Consent to being told something is complete defense to defamation. Here, proprietor specifically asked and received information that might otherwise be defamatory.

A defendant was charged with sale and distribution of illegal drugs in violation of the state criminal code. The trial took place in the county in which most of the illegal sales occurred. The defendant was convicted and, based on the prosecutor's recommendation, received a sentence of 18 months from the judge. A prosecutor in a neighboring county who was running for reelection on a law and order platform publicly denounced the sentence. That prosecutor obtained a grand jury indictment against the defendant for sale and distribution of illegal drugs to juveniles, some of which had occurred in that county. This offense carried more severe penalties and required the additional element that the defendant knew drugs were being sold to juveniles. Will the defendant be successful in a motion to dismiss the indictment?

Counties are not separte sovereigns, and as such cannot be tried for same offense once jeopardy attaches. Jeopardy attaches to a lesser included offense and prevents another trial for the greater offense. Here, the greater offense was the sale of drugs to juveniles. Once convicted of the lower offense, jeopardy barred the greater offense. "Under the Blockburger rule, two crimes do not constitute the same offense if each crime requires proof of an additional element that the other crime does not require, even though some of the same facts may be necessary to prove both crimes. Here, however, the illegal drug sale charge is a lesser included offense of the sale to juveniles offense, and attachment of jeopardy for a lesser included offense generally bars retrial for the greater offense."

A town with a population of 30,000 merged with a city of 60,000. To protect voting rights of the citizens of the former town, a proposal was made that for a period of 20 years, beginning at the date of the merger, the city council of the merged city would consist of six persons. Each formerly separate municipality would be divided into three council districts. Each district from the former town would have approximately 10,000 residents, and each district from the former city would have 20,000 residents. A mayor would be elected at large. Before this proposal was placed on the ballot, the state attorney general issued an advisory opinion stating that the proposal was not in violation of any state statutory or constitutional provision. The proposal was placed on the ballot and was carried by large majorities in both the town and the city, and the districts were carved out. Three taxpayers filed suit to enjoin the holding of an election with council districts of such disparate proportions. The suit reached the state supreme court, which ruled that the governmental formula was constitutional under both the state and United States Constitutions. The plaintiffs wish to take the case to the United States Supreme Court. How should the Supreme Court proceed?

Court can hear the federal issues involved but decline to rule on state issues. B) is incorrect even though the state supreme court may have had an independent state ground for finding the law constitutional under its state constitution. The Supreme Court will refuse to hear the case only if the state ground is adequate by itself to support the decision as well as independent, so that the Court's review of the federal ground for the decision would have no effect on the outcome of the case (such as if the state court had found the law invalid under both the state and federal Constitutions). Here, the Supreme Court's review of the state court opinion on the law's federal constitutional status may have an outcome on the case regardless of the state court's decision on the state constitutional issue; the Court therefore will hear the federal issues involved

An investor financed the purchase of a parcel of land with a $500,000 loan from a bank, secured by a mortgage on the land. After an unsuccessful search to find a developer to subdivide and build homes on the land, the investor leased the existing home to a tenant for five years. During the fourth year of the lease, the investor defaulted on the bank loan. The bank brought a foreclosure action, joining the investor and the tenant in the proceeding. The land was sold to a buyer at the foreclosure sale for the outstanding amount of the loan. After the buyer obtained a judgment in an ejectment action against the tenant, the tenant brought an appropriate action for damages against the investor. As between the tenant and the investor, for whom should the court rule?

Covenant of quiet enjoyment implies that any renter will not be either constructively or actually evicted from the premises by someone with paramount title for the term of their lease.

A landowner embarked on an expedition into a remote jungle, leaving no means to communicate with him. Because property values suddenly began plummeting in the landowner's neighborhood, his son believed that it was imperative to sell his father's property before it became worthless. Having no way to speak to his father ahead of time, the son prepared a deed conveying the property to a buyer, but left the line for the buyer's name blank. He then signed his father's name on it as the grantor, and handed the deed to the buyer. The deed, however, did not include any language regarding the amount the father was to receive in exchange for the property. The buyer believed that the son was the owner of the property. When the father returned, he was happy that the property had been sold. If the buyer changed his mind and now wishes to have the conveyance set aside, which of the following would be his best argument?

Deed here isn't valid because son signed it, and son did not own the land. Deeds that are forged are void. Even though buyer was not named in the deed, courts presume buyers can fill in their own name on the deed upon acceptance.

A consumer purchased a luxury automobile from a dealer on credit. After the consumer failed to make a number of the required payments, the dealer filed a civil action against the consumer in a federal district court to recover the balance due on the account. The dealer properly served process on the consumer. Several months passed, and the consumer did not file any response to the complaint. The dealer then filed a motion asking the clerk of court to make an entry of default, and the clerk did so. Without notice to the consumer, the dealer filed a motion for default judgment, which the court entered. When the consumer learned a month later that a default judgment had been entered against him, he immediately filed a motion for relief from judgment under Federal Rule of Civil Procedure 60. If the court grants the consumer's motion, what is the most likely reason?

Defense to a default judgment is that there was excusable neglect and they have a meritorious defense. "Once a default judgment has been entered, it may be set aside on any of the limited grounds listed in Federal Rule 60 applying to all judgments. On motion and just terms, the court may relieve a party from a final judgment or order on the following grounds: (i) mistake, inadvertence, surprise, or excusable neglect; (ii) newly discovered evidence that by due diligence could not have been discovered in time to move for a new trial; (iii) fraud, misrepresentation, or other misconduct of an adverse party; (iv) the judgment is void; (v) the judgment has been satisfied, released, or discharged; a prior judgment on which it is based has been reversed or otherwise vacated; or it is no longer equitable that the judgment should have prospective application; or (vi) any other reason justifying relief from the operation of the judgment.'

A restaurant owner in State A bought two large freezers from a manufacturer of commercial refrigeration equipment with its principal place of business in State B. Within one week and after being fully stocked with meat, one of the freezers broke down. The restaurant owner filed a state-based products liability action against the manufacturer in federal court in State A, and included a demand for a jury trial. Under the law in State A, jury verdicts do not need to be unanimous, but the Federal Rules of Civil Procedure require jury verdicts to be unanimous. At trial, the restaurant owner makes a motion asking the court to apply the State A law. How should the court rule on the motion?

Deny. Federal procedural rule on point supersedes any conflicting state procedural law! 5 types of substantive laws: Conflict (or choice) of law rules; • Elements of a claim or defense; • Statutes of limitations; • Rules for tolling statutes of limitations; and • The standard for granting a new trial because the jury's damages award was excessive or inadequate.

A developer constructed several small stores in a commercial district. She received a bid from a contractor to install awnings on the front windows of the stores. The developer had heard that the contractor did shoddy work, but the price was right and the contractor expressly assumed all of the risk of any liability. The developer subsequently sold one of the stores to a barber. A few months later, an awning collapsed without warning, injuring a customer who was about to enter the barbershop. An investigation by the building inspector revealed that the awning collapsed because the brackets used by the contractor were cheaper and weaker than the required brackets, although they looked the same. The developer and the contractor are now both bankrupt. If the customer sues the barber for his injuries, is the customer likely to prevail?

Even though an owner has a duty to maintain the premises (including the areas adjacent), cannot be liable for installations of things they had no ability to oversee and that appear to be fine.

An employee worked as a third-shift supervisor at a manufacturing plant. One of his duties was to ensure that all timekeeping records accurately reflected the time his crew actually worked. Workers, including the employee, were then paid for whatever hours the timecards reflected. The employee was also required to assist in submitting budgets for payroll. Needing to leave work early for a second job that he obtained, the employee had one of his trusted co-workers punch his card out at the regular time every day of the week. At the end of the week, he signed the timecard with those hours included, and was paid accordingly. He continued to do this for several weeks before being discovered. What crime has the employee committed?

False pretenses. Employee here At common law, theft by false pretenses occurs when a defendant (i) obtains title; (ii) to the property of another; (iii) by an intentional (or knowing) false statement of past or existing fact; (iv) with the intent to defraud another. In the instant case, the employee's conduct meets all of the elements of the crime.

A single man with a life insurance policy that pays his designated beneficiary $70,000 upon his death was killed in a car accident. His former girlfriend, a resident of State A, was named as beneficiary, but his mother, a resident of State B, also filed a claim for the life insurance proceeds. The insurance company, a State C corporation having its principal place of business in State B, filed an interpleader action in federal court to protect itself from potentially multiple and inconsistent claims. May the insurance company bring the interpleader action in federal court?

Federal interpleader Act allows for interpleader to bring in any two claimants if they are citizens of different states and the amount in controversy is more than 500. This was not covered in lecture or in any outlines, nice.

A city has a permanent display of passages from various political and religious documents on plaques along the side of its municipal building, including the Declaration of Independence, the Ten Commandments, and the lyrics to "The Star-Spangled Banner." The city has added the documents over time when they have been donated by various civic groups. Members of a religious organization that worships the planet Mars donated to the city a plaque containing their basic creed and requested that the plaque be displayed along with the other plaques on the side of the municipal building. The city refused to display the plaque, and the organization that donated the plaque brought suit against the city, claiming that its rights to freedom of speech were being violated by the city's refusal to include the plaque along with the others. How should the court rule?

For the city. Gov has right to pick and choose what messages it want's to convey when speaking.

Joint Tenancy: Creation

Four Unities - "T-TIP" - Joint tenants must take their interests: T - At the same time T - Same title I - Identical interests and P - Possess the whole

While traveling on a commercial bus line, a passenger was injured when some luggage fell on him. As required by applicable state law, the bus company's in-house attorney conducted an investigation and filed the required report with the state transportation department. The passenger subsequently filed a civil action against the bus company in federal district court, seeking compensatory damages for the injuries he suffered. During discovery, the passenger's lawyer served on the bus company a request for production of documents, including a request for the report that the bus company filed with the state. The bus company objected to the request for the report and refused to produce it on the grounds that the report was privileged and protected from discovery under the work product doctrine. It did, however, produce other documents that were requested. The passenger then filed a motion to compel production of the report. If the court finds that the bus company's claims of privilege and work product were not substantially justified, what orders must the court make relating to the passenger's request for production of the report?

Good faith effort to resolve the issue prior to motion to compel.

A bartender and a patron were arguing over an upcoming football game. Their argument grew heated, and the bartender reached over the bar and pushed the patron's shoulder. Enraged, the patron picked up a beer bottle, smashed it, and lunged over the bar. He pinned the bartender against the wall and swung the broken glass toward the bartender's throat. The bartender managed to grab a baseball bat he kept hidden under the bar and hit the patron, causing the patron to fall and strike his head on the edge of a table. The patron died as a result and the bartender was charged with manslaughter in a jurisdiction that recognizes the "imperfect self-defense" doctrine. Should the bartender's claim of self-defense succeed?

If person escalates a minor conflict into deadly conflict, defendant can respond using deadly force even though they may have started a fight. This is a defense to manslaughter.

The owner of a custom jewelry supply shop placed an order with a manufacturer for 500 pairs of sterling silver "posts" of the type that are used to make pierced earrings. However, when the manufacturer started to fill the order, it had only 450 pairs of sterling silver posts available. The manufacturer shipped the 450 pairs of sterling silver posts to the shop owner, plus 50 pairs of higher-priced 10-karat gold posts, without making any adjustment in price. The manufacturer enclosed a note with the order, explaining to the shop owner that it was sending the last of the sterling silver posts in stock, plus the 50 10-karat gold posts to accommodate the buyer. Did the manufacturer's shipment constitute an acceptance of the shop owner's offer?

If seller sends goods with note explaining the deficiency of the order, it is a counteroffer not an acceptance! If seller simply sends wrong goods with no explanation, it is an acceptance and a breach under Article 2.

A professional athlete was involved in a car accident after leaving his team's stadium following a game, and a child was killed. The child's parents brought a wrongful death lawsuit against the athlete, which eventually was settled. A reporter for a sports gossip website accurately reported that, as part of the legal settlement, the athlete was required to seek counseling for alcoholism. The website reporter incorrectly believed that she had received this information from a press release by the athlete's agent, but in fact it had been supplied by an unauthorized source, and no public information was released of the terms of the settlement. The athlete brought a privacy action against the website reporter based on the public disclosure of private facts, but the jury rejected his claim. What is the likely explanation?

If the website reporter prevails, it will be because the publication involved a matter of public interest. The invasion of privacy action based on public disclosure of private facts requires the following elements: (i) publication or public disclosure by defendant of private information about the plaintiff, and (ii) the matter made public is such that its disclosure would be highly offensive to a reasonable person. However, publication in such a case may be privileged if the matter is one of legitimate public interest, as long as it is made without actual malice (i.e., knowledge of falsity or reckless disregard for the truth).

An elderly woman entered into a contract with a company in the business of providing home care services. Believing that she had been duped by representatives of the company, the woman commenced an action in federal court, properly based on diversity, seeking rescission of the contract. The company answered, denying the principal allegations of the woman's complaint and asserting a counterclaim against the woman for breach of contract. In addition, the company timely served a demand for a jury trial. The woman did not. Which statement best describes the roles of the judge and jury as finders of fact in the trial of the parties' claims?

Jury will first hear issues relating to breach of contract claim. If cases present both legal and equitable questions, the jury will first hear the legal questions and their finding of fact will bind the court on the matter of equity for the next question.

A landowner died, validly devising her land "to my son for life, then to my brother and sister in fee simple." Without obtaining the brother and sister's consent, the son borrowed $20,000 from a bank, secured by a mortgage on the land, to make improvements to the land. Five years later, the son died. The brother and sister took possession of the land, but failed to make any mortgage payments. If the bank sues to recover the delinquent payments, for whom should the court render judgment?

Life tenant cannot do anything to encumber the land without permission of the remaindermen. Here, Life tenant took out debt that was not permitted. As such, remaindermen take the property free of the debt.

The plaintiff and defendant were in a two-car accident. The plaintiff filed a negligence action against the defendant in federal district court, seeking compensatory damages. After the accident but before the action was filed, the defendant's attorney spent $25,000 in investigation fees to determine the identity of and track down a particular eyewitness to the accident. He did so exclusively to get ready for the anticipated litigation. Must the defendant reveal the identity of the eyewitness to the plaintiff in discovery? Press Enter or Space to submit the answer

Must disclose the identity only if the defendant might call the eyewitness at trial or if plaintiff requests their identity. As part of required initial disclosures, a party must provide the identities of individuals likely to have discoverable information that the disclosing party may use to support its claims or defenses.

A landowner granted to his adjoining neighbor a written easement in a driveway that crosses the southwest corner of the landowner's property. The easement was not recorded. A statute of the jurisdiction in which the landowner's and neighbor's properties are located provides: "No unrecorded interest in real property shall be good against subsequent purchasers for value without notice unless the conveyance is recorded." In which of the following cases has the neighbor's easement been terminated?

Nieghbor told the landowner she will no longer be using the driveway. Easement by estoppel here. For an easement to be extinguished by estoppel, there must be (i) some conduct or assertion by the owner of the easement, (ii) a reasonable reliance by the owner of the servient tenement, coupled with (iii) a change of position. Remember ENDCRAMP for termination of easements.

At the defendant's trial for murder, facts were introduced that the defendant acted in the heat of passion. After a lengthy trial, the defendant was convicted of manslaughter. On appeal, the conviction was reversed on procedural grounds. The state immediately moved to retry the defendant, again bringing murder charges against her. The defendant moved to strike the murder charge, and the court refused to grant the motion. After the second trial, the defendant was again convicted of the lesser charge of manslaughter. The defendant appeals the second conviction, claiming that it violated her constitutional rights. May the second conviction stand?

No, cannot retry a person for an offense more serious for that than which they were convicted! Here, defendant was convicted of manslaughter, so could not be retried for murder.

The plaintiff sued the defendant, alleging that the defendant allowed her dogs to roam onto the plaintiff's land and cause significant damage to his landscaping. The defendant denied the allegations and called a witness to testify on her behalf. The witness testified on direct examination that she visited the defendant every day and that the defendant never allowed her dogs to leave the perimeter of her property. On cross-examination, the plaintiff's counsel presented the witness with a letter written by the witness to a friend in which she expressed her dismay that the defendant allowed her dogs to roam throughout the neighborhood. The plaintiff's counsel requests that the witness read the letter to herself prior to cross-examination. The defendant objects. May the witness refer to the letter?

No. Cannot refer to the letter unless the witness claims an inability to remember first!

A patient properly filed a medical malpractice claim against a doctor in federal district court. The complaint simply asserted negligence as the grounds for relief without any facts supporting the claim. The doctor filed a pretrial motion for a more definite statement, which the court denied. Immediately thereafter, and without submitting an answer, the doctor filed a motion to dismiss, asserting that the court lacked personal jurisdiction. Will the court grant this motion to dismiss? Press Enter or Space to submit the answer

No. Defendant has to file motion to dismiss based on PJ pursuant to 12(b)(6) either in answer or motion, whichever happens first.

A state set up an intrastate message routing system to carry messages to and from the various state agency offices located throughout the state. This proved to be cheaper and more efficient than the United States Postal Service. The message service worked so well that the state offered the messenger service to its employees as a fringe benefit. Moreover, it expanded delivery options beyond state offices to any address in the state and permitted the employees to use the service for personal correspondence as well as for official business. Are the state's actions constitutional?

No. Federal Postal Service has monopoly and no other state agency can compete.

Profit

Nonpossessory interest in land, allowing the grantee to enter on the land and remove resources of the land. Can be conveyed by grantee to a third-party.

The owner of a store in a small beach town contacted a manufacturer about buying 100 Adirondack chairs. The parties reduced their agreement to sell and buy the 100 chairs to a signed writing that contained all of the essential terms, as well as a merger clause. When the chairs were delivered to the store owner, she discovered that they were not "Adirondack chairs," as she understood that term and as that term was commonly used, but a different style of chair altogether that was manufactured in the Adirondack region of New York. The store owner refused delivery, and the manufacturer sued the store owner for breach of contract, arguing that he complied with the terms of the contract. Will the store owner be allowed to introduce evidence of the meaning of the term "Adirondack chair"?

Parol evidence can be introduced to clarify ambiguous terms in a writing even after a final writing has been signed! Here, legitimate ambiguity as to what the term "Adirondack" meant to each party.

Following their divorce, the plaintiff and defendant were engaged in a bitter custody battle over their two children. The plaintiff is seeking to testify as to statements made by the defendant prior to the divorce in which she had told him that she did not feel that she could properly care for the children by herself. The defendant objects, alleging that the statements are privileged as a confidential communication made during the marriage.

Privilege does not apply here because the case is between the two spouses. In any other situation, confidential communications would have applied!

A consumer bit into a hamburger at a restaurant and cut her gum on a piece of bone in the meat. The bone was over an inch long. The consumer sued the meat processor in strict liability and negligence. At trial, she presented evidence that the processor supplied the ground beef that the restaurant used to make its hamburgers and that she was injured from the piece of bone in the meat. The meat processor presented evidence that restaurant employees prepared the hamburger patties by hand from the ground beef supplied by the processor, and asserted that one of the employees would have found the piece of bone had they made a reasonable inspection of the meat while preparing it. At the close of the evidence, the meat processor moved for a directed verdict on the consumer's negligence claim. Based on the facts above, how should the court rule?

Res ipsa loquitor can apply here! Negligence can be implied. Jury could find that a bone in the meat is negligence that could not have occurred without the negligence on the part of the meat processor. Important to note that, even if restaurant was negligent in not discovering the bone, this does not cut off liability. Intermediaries failure to discover a defect is not a superseding cause!

A radio station was sued by a former program host for defamation. The station moved to dismiss the complaint for failure to state a claim and for judgment on the pleadings. In response, the host voluntarily dismissed the action and filed a new action, alleging the same claims but also addressing the pleading defects raised by the station. The station moved again for judgment on the pleadings and accompanied the motion with affidavits asserting that the alleged defamatory statements were true. The host's attorney should respond to the new motion by doing which of the following?

Should have requested additional time to respond to the allegations. When a party includes allegations and affidavits on the pleadings, judge can treat it like motion for summary judgment. Party opposing the motion may present reasons why they need more time to submit affidavits of their own or obtain depositions.

A State A citizen and a State B citizen were in a car accident in State C. The State A citizen filed a negligence action against the State B citizen in a State C state court, seeking $500,000 in damages. If the State B citizen wishes to remove the action to federal district court, in which federal district should the State B citizen file a notice of removal?

State C court! Removal is proper in the state court that encompasses the action. Here, that would be State C.

A township located in a farming community was composed mostly of persons belonging to a specific religious sect. To help instill proper respect for authority in children, which was a central tenet of the sect, and to maintain order in the classroom, the local school board allowed teachers to inflict corporal punishment. Such punishment was inflicted on a fourth grader in a township school immediately after his teacher saw him pulling a girl's hair. Neither he nor his parents belonged to the religious sect. When the boy's parents learned of the incident, they hired an attorney. Rather than suing the teacher for battery as permitted under state law, the attorney brought an action against the teacher under a federal statute providing a cause of action for damages against any government employee who deprives a person of his constitutional rights. Should the court find the policy allowing corporal punishment to be constitutional? Press Enter or Space to submit the answer

Stupid question. Apparently the policy should be allowed because paddling is not unconstitutional under the 8th and 14th Amendment.

An owner purchased a parcel of property adjoining a five-foot-wide strip, which was a private right-of-way. Unsure where the exact boundaries of her property were located, the owner planted a garden on the five-foot right-of-way strip and enclosed it with a wire fence two weeks after taking up occupancy. The owner maintained the fence and garden for 20 years, at which time she removed the fence and smoothed out the ground where the garden had been located. Five years later, the owner entered into a written contract to sell the property to a buyer. The description in the contract included the five-foot strip. After research in the county recorder's office, the buyer discovered that the strip was a private right-of-way when the owner purchased the property. After properly notifying the owner of the problem prior to closing, the buyer refused to tender the purchase money to the owner when the closing day arrived. The owner sued the buyer for specific performance of the real estate sales contract. The jurisdiction's statutory adverse possession period is 15 years.

The buyer prevails. NEED QUIET TITLE ACTION TO MAKE TITLE WITH ADVERSE PROPERTY MARKETABLE.

A defendant was charged with rape after a colleague accused him of assaulting her after a company event. The defendant admits that they had a sexual encounter, but claims that it was consensual. At trial, which of the following is most likely to be admitted in the defendant's defense?

The defendant had colleague had consensual sex before. Evidence of alleged victim's sexual past usually not allowed, but in criminal case specific instances of sex between the victim and the defendant can be shown as a defense to rape.

A tenant had been leasing an apartment from a landlord for more than 10 years. There was no written lease; however, the parties had agreed orally, at the beginning of the rental, what the rent would be per month. The tenant left a check for each month's rent in the landlord's mailbox on the first day of that month, without fail. On September 10, the landlord handed the tenant a handwritten note stating that the lease was to be terminated effective that October 1. On October 1, the tenant placed a check for the rent in the landlord's mailbox, and the landlord brought an action for unlawful detainer against the tenant. Who is likely to prevail?

The tenant. Periodic tenancy created when no express terms exist, and the tenancy can then take on the form of whatever payment system they decide on. For a tenancy that is less than a year, need the full duration of the tenancy agreement to terminate. Here, that would be one month.

An astronomer who had studied the moon for more than 20 years became convinced that the moon began sending him messages revealing how to achieve an afterlife. The astronomer created a website where he shared the secrets that the moon had revealed. A wealthy follower funded the construction of an observatory to serve as the national headquarters for the astronomer and his followers. The astronomer continued to study the moon from the observatory and held weekly meetings at the observatory at which he revealed the continuing messages that he received from the moon. Some followers attended the meetings regularly, and some watched the meetings via an Internet feed. After receiving his first state property tax bill for the observatory, the astronomer applied for a state property tax exemption that was available for houses of worship. After conducting an investigation, the state denied the observatory tax-exempt status. The astronomer filed suit to overturn the denial. The state moved for a summary judgment, arguing that the exemption is available only for houses of worship and that, because the astronomer and his followers do not belong to a recognized religion, the observatory is not eligible for the exemption as a matter of law. How should the court rule on the motion?

They should be allowed to prove that their beliefs are actual religious. Can question sincerity of the beliefs. If beliefs determined to be sincere, cannot question the truthfulness of the belief!

Transferability of Easement Appurtenant

Transfers with dominant estate automatically, with servient estate as well unless buyer is BFP and had no knowledge of the easement.

A woman owed her auto mechanic $2,000. The debt remained unpaid until any claim for its repayment became barred by the statute of limitations. The woman then agreed in writing to pay the mechanic $1,500, but failed to pay him. If the mechanic sues the woman for the $1,500, will the court rule in his favor?

Yes because the agreement was made in writing. If technical defense arises to past obligation, here debt, either part performance or a writing will make the payment enforceable even after the technical defense (here statute of limitations) arises.

Congress declared war on a nation in Asia. Congress also passed a statute making it a crime to make public statements in support of the Asian nation and against the United States. A United States citizen subsequently spoke out against the war at a rally in front of a federal building. During her speech, the citizen urged people to "smash the windows of the federal building like U.S. troops are doing" in the Asian nation. Several members of the frenzied crowd did as the speaker urged. The speaker was immediately arrested and charged with violating the statute. Can the speaker successfully defend by asserting that the statute violates her First Amendment speech rights? Press Enter or Space to submit the answer

Yes, because the statute is overbroad. When a statute that deals with free speech is overbroad, even a person engaging in speech that might be incitement cannot be charged because the entire statute is facially invalid.

A utility company constructed a building costing approximately $2 million that encroached on a rancher's property. The rancher is suing the company in federal district court to force the company to remove the office building. In the same action, the rancher is asking for $200,000 in damages incurred because of the trespass. The court has diversity of citizenship jurisdiction. Is the company entitled to a jury trial?

Yes, only for issues regarding the trespass. FUNDAMENTAL RIGHT for jury trial in cases regarding issues of fact. If seeking equitable relief, can be satisfied through a bench trial. Here, trespass action merited jury. Judge could decide the issues of the company removing the office building.

The driver of a car filed a civil action in federal district court against the owner and operator of a truck that collided with her vehicle. The driver has reason to believe that the truck was serviced at a particular auto repair shop shortly before the accident. The driver wants to see any records the repair shop has regarding the truck's servicing because a failure to properly service the truck could help prove the truck owner's liability. The driver requested the service records from the truck owner, but the truck owner has no service records in his possession. Can the driver obtain the truck's service records from the repair shop?

Yes. Driver can obtain subpoena even on non-party if they have relevant material.

A homeowner who regularly borrowed garden tools from his neighbor went to the neighbor's house to borrow the neighbor's leaf blower. The neighbor was not at home, but the leaf blower was in his unlocked garage with his other garden tools, and so the homeowner took it. Unbeknownst to the homeowner, the neighbor had drained the oil from the leaf blower's motor. The homeowner ran the leaf blower for an hour; the motor was totally destroyed because it had no oil. The value of the leaf blower at the time that the homeowner took it was $300. An identical new leaf blower costs $500. The cost of repairing the motor is $150. A new motor will cost $250. If the neighbor sues the homeowner on a theory of conversion and is successful, what damages can he recover?

300, because that was fair market value AT THE TIME OF THE CONVERSION. If the price raises or lowers after conversion, it is irrelevant.

A buyer of a new car owed the car dealership where she purchased the vehicle $1,000 on a promissory note that was due on December 30. The buyer determined that she would be unable to pay the note on its due date, and she informed the owner of the dealership of that fact. The owner told her that she would not have to pay the debt if she bought him four tickets to a popular concert on January 15 that had been sold out for weeks, because she worked as publicist for the concert venue. She agreed to do so, and the parties memorialized their agreement in a signed writing on December 18. On January 2, the dealership filed suit against the car buyer for failure to pay the $1,000 promissory note, before the car buyer had secured the concert tickets for the owner. May the car buyer have this action enjoined by introducing evidence of the December 18 agreement?

Accord agreement (changing method of performance) suspends the original contract. It does not discharge the original contract, just suspends it. As such, until it is clear accord will not happen, the debtor can enjoin any attempt to enforce the terms of the original contract using the accord as an equitable defense. Once accord carried out, the original contract is discharged as well.

An engineer licensed by the state was the principal design engineer for a wastewater treatment plant's aeration system. Detailed recommendations for designing aeration systems for this type of plant had been published by a panel of engineers after lengthy study. The engineer fully complied with the recommendations in his design. Nevertheless, the treatment plant's aeration system suffered a major failure, causing the release of bacteria-laden water into a river that damaged a fish hatchery run by the plaintiff. If the plaintiff sues the engineer and prevails, what is the likely explanation?

Because the engineer knew of a better design that would have prevented the failure. A person who is a professional is required to exercise such superior judgment, skill, and knowledge as he actually possesses. If the engineer knew of and could have used a better design for the aeration system, he has breached his duty as a professional. NOT ABNORMALLY DANGEROUS ACTIVITY. An activity is characterized as abnormally dangerous only if it involves a substantial risk of serious harm to persons or property, even when reasonable care is exercised by all actors, and the activity is not a matter of common usage in the community.

A homeowner contracted with a local builder to build a wooden deck onto the back of her house. The contract called for half of the contract price of $20,000 to be paid to the contractor before he began work and the other half to be paid to him when the job was completed. The contractor began the work but, partway through the job, he got an offer for a rush job that paid better and abruptly quit. The homeowner sues the contractor for specific performance. Will she prevail?

CANNOT COMPEL SPECIFIC PERFORMANCE OF PERSONAL SERVICES - TANTAMOUNT TO INVOLUNTARY SERVITUDE

A tourist from State A was severely injured in a bar fight in State B. The tourist filed a battery action against one of the bar's patrons, seeking $100,000 for his injuries. The defendant patron claims that the tourist is mistaken about who hit him. The patron says that he did not touch the tourist. The patron claims that it was the bar's bouncer-who looks like the patron-who hit the tourist and then continued to pummel him. The bouncer claims that he never touched or harmed the tourist. Can the patron assert a third-party claim against the bouncer to bring him into the action?

Cannot implead a third party defendant unless seeking some type of indemnity or contribution for the liability you owe to a plaintiff. Here, the patron just totally denies liability and is not trying to recover liability, he cannot assert a third-party claim. HAS NO LEGAL STANDING. D is rule for counter-claims not impleader!!!

A gang member threatened to kill the defendant unless he robbed a convenience store and gave the proceeds to the gang member. The gang member also demanded at gunpoint that the defendant kill the clerk to prevent identification. In abject fear of his life, the defendant did everything that the gang member requested. If the defendant is arrested and charged with murder and robbery in a common law jurisdiction, what result?

Defendant should be acquitted of robbery but convicted of murder. Duress is a defense to every crime except intentional homicide. Can be convicted of murder even if forced to do it.

A wholesaler who sold hair-care products to beauty salons and spas contracted to purchase 20 hand-held ionic hair dryers to be delivered no later than May 1. She paid for the hair dryers at the time she placed her order. When the hair dryers were delivered on May 1, the wholesaler noticed that they were hot-air, and not ionic, hair dryers. She immediately contacted the manufacturer, which refused to grant the wholesaler any remedy. If the wholesaler decides to keep the hair dryers and brings a claim for breach of contract against the manufacturer, which of the following awards is most likely?

Difference between value of ionic hair dryers and value of the hot air-dryers. If a buyer accepts non-conforming goods under the UCC there is still a breach and the buyer has the right to sue for damages based on the difference of the value of what was given as opposed to what was actually ordered. WARRANTY DAMAGES. Can also get incidental and consequential damages.

A young woman who recently graduated from college landed her dream job teaching kindergarten at the same elementary school she had attended as a child. The young woman's contract provided that she would be paid $40,000 for the school year, and that she could be fired only for just cause. Days before the young woman was to begin teaching, the school's principal fired her, without cause, so that he could hire his cousin for the job instead. The young woman submitted her resume to an employment agency, but was so depressed over the loss of her dream job that she turned down a nearby school's offer for a similar job paying a $30,000 salary, and instead spent the rest of the school year miserably unemployed. In an action by the young woman against the elementary school for damages, which of the following awards is most likely? Press Enter or Space to submit the answer

Duty to mitigate if reasonable. Here, could have easily and reasonably mitigated damages by getting another job worth 30k. Instead, chose not to. Only entitled to 10k because of that other job she could have taken.

A state statute prohibited the sale or possession of any food product containing more than one part per billion of a dangerous pesticide. An out-of-state driver taking her recreational vehicle through a corner of the state was stopped at a state inspection station. When the state trooper learned that the pantry of her RV was stocked with food, he asked to test a few samples of her baked goods. The samples contained about 600 parts per billion of the prohibited pesticide, and all of the other baked goods in her possession were tested and found to have the same level of pesticide. All of her baked goods, worth about $150, were confiscated and destroyed. The state in which the driver lived has no laws governing the pesticide level of baked goods. A federal law designed to protect agricultural workers requires that any food product containing more than 500 parts per billion of the toxic pesticide must be labeled as such and be in special containers. The driver brings an action in federal court asserting that the state statute is invalid because it is preempted by the federal law. How should the court rule as to this claim?

For the state -- the state law is not preempted by the federal law because the purposes of the federal law are different from the challenged statute. If there was actual conflict, state law would have been preempted. STATE LAWS CAN BE MORE STRICT THAN FEDERAL LAWS, NOT MORE LENIENT UNLESS CONGRESS INTENDS OTHERWISE

To encourage minority business and foster pride in minority heritage, a state adopted legislation exempting magazines and other periodicals from the state's receipts tax if 20% of the magazine is devoted to articles concerning minorities (a commission was set up to sample magazines to determine on a yearly basis whether they should be exempt). A publisher produced a sports magazine in the state that occasionally contained articles about minority athletes, but the commission determined that the publisher's magazine was not eligible for the receipts tax exemption. After paying the tax assessed on her magazine, the publisher sued for a refund. How will the court most likely rule?

In favor of the publisher, because the tax violates the First Amendment freedoms of speech and press. Cannot single out the press with a single regulation or tax unless there is a compelling interest.

At trial, questions have been raised as to whether the proposed testimony of a witness concerning an out-of-court statement falls within the present sense impression exception to the hearsay rule. The proponent wants to introduce evidence demonstrating that the proposed testimony does fall within the exception. How should a preliminary determination of the admissibility of the witness's testimony be made?

Judge should determine if it is a present sense impression, and can consider any nonprivileged evidence in doing so, even if it wouldn't be admissible at trial. "The judge determines whether the testimony falls within an exception to the hearsay rule, and is generally not limited by the rules of evidence in making that determination. The Federal Rules of Evidence distinguish between preliminary facts to be decided by the jury, which involve whether the proffered evidence is relevant, and preliminary facts decided by the judge, which involve whether the evidence is competent, that is, not barred by an exclusionary rule. All preliminary fact questions that determine the applicability of an exception to the hearsay rule must be determined by the judge, because the competency of the evidence will depend on that preliminary fact determination. In making this preliminary fact determination, the trial court may consider any nonprivileged relevant evidence, even if such evidence would not be admissible at trial."

On April 10, the owner of a small farm mailed a letter to a new resident of the area who had expressed an interest in buying the farm. In this letter, the farm owner offered to sell the farm to the resident for $100,000. The offer expressly stated that the offer expires on June 1, "if acceptance by the offeree has not been received by the offeror on or before that date." On the morning of June 1, the resident sent a written acceptance to the farm owner by messenger. However, through negligence of the messenger company, the acceptance was not delivered to the farm owner until June 2. On June 4, the farm owner entered into a contract to sell the farm to another buyer for more money but did not inform the resident of the transaction. When the resident followed up by phone on June 10, the farm owner told him that he had sold the farm to another buyer. Which of the following is the most correct statement?

Mailbox rule (that acceptance is valid when mailed) does not apply here because there is express condition in the contract requiring receipt of the acceptance before the contract is valid.

A pedestrian was struck and seriously injured by a car driven by an intoxicated driver. The driver had been served several alcoholic drinks by a bartender at a local bar. The pedestrian sued the bartender in a jurisdiction that does not have a dramshop act. Is the bartender vicariously liable for the pedestrian's injuries?

No because no dramshop act in this jurisdiciton. Dramshop act imposes vicarious liability on bartenders in some instances.

Pursuant to a contract, a landscaper performed $30,000 of landscape work for a homeowner. By coincidence, the homeowner and the landscaper were involved in an automobile accident that was unrelated to the landscape work. The homeowner was injured in the accident and sued the landscaper in federal district court for negligence, seeking $100,000 in damages. The homeowner and the landscaper are citizens of different states. May the landscaper assert and maintain a counterclaim against the homeowner for breach of contract, seeking the $30,000 due under the landscape contract?

No because this is a permissive counterclaim (not resulting out of same transaction) and as such would need SMJ, either FQ or diversity. Here, neither of those present and as such cannot be brought as counterclaim but could sue him separately in state court.

On the last play of a playoff football game, a game-winning touchdown was nullified by a questionable penalty called by the referee. To register her displeasure but without intending to hit anyone, a fan sitting in the stands threw a bottle onto the field that just missed the head of the referee, who was looking in the other direction and did not see the bottle being thrown. The fan was charged with assault. Should the fan be convicted?

No. Assault is a specific-intent crime, must have the requisite intent to actually commit the crime to be convicted. To distinguish between criminal and tortious assault, look for words like CONVICTED in fact pattern. Both still require intent.

A father told his adult daughter that if she gave up smoking for the next 12 months, at the end of that time he would give her $10,000. She agreed to stop smoking, but later that day had doubts about whether her father would actually pay up if she complied. She contacted her stepmother, who told her to go ahead and quit smoking, and she would make good on the father's promise to pay her if he refused to do so. That very day, the daughter quit smoking and never smoked again. Eleven months after his conversation with his daughter, the father died. One month later, the daughter sought payment of the $10,000 from her father's estate, which refused to pay. The daughter then asked her stepmother for the $10,000 but the stepmother also refused to pay. The daughter filed a claim against her stepmother for $10,000. She proves at trial that she has submitted a claim for $10,000 to the executor of her father's estate and has been refused payment. What is the best argument for the court's rejecting this claim against her stepmother?

Promises to pay the debt of another, here the stepmom's promise to pay on the father's behalf, MUST BE IN WRITING UNDER STATUTE OF FRAUDS

A state statute has adopted the common law definition of larceny. Another statute provided as follows: "It shall be an affirmative defense to a crime if the defendant establishes by clear and convincing evidence that, due to a mental disease or defect, he was unable to appreciate the criminality of his conduct or conform his conduct to the requirements of the law." A homeowner was leaving town for two weeks and he asked his neighbor to stop by the house each day and water the plants. While at the homeowner's home, the neighbor found the keys to the homeowner's new car. The neighbor took the car and drove it into town to show his friends. The neighbor told all of his friends that he had purchased the car. The homeowner returned home three days early, saw that the car was missing, and called the police. Later that day, the neighbor was arrested and charged with larceny. At the neighbor's trial, the neighbor testified that he intended to return the car. Additionally, two psychiatrists testified that, due to a mental defect, the neighbor suffered from an extreme inferiority complex and delusions of grandeur. The doctors further testified that his mental condition caused him to take the car and to tell other people that he owned it. At the conclusion of the evidence, the court's instructions to the jury included the following: 1. If you find by a preponderance of the evidence that the defendant intended to return the car, you should find the defendant not guilty. 2. If you find by a preponderance of the evidence that, due to a mental disease or defect, the defendant was unable to appreciate the criminality of his conduct or conform his conduct to the requirements of the law, you should find the defendant not guilty. The neighbor was found guilty and he appealed, claiming that the jury instructions violated his constitutional rights. How should the appellate court rule?

The first instruction is wrong. State must prove every element of an offense -- they cannot shift the burden onto the defendant. This instruction does just that and, as such, is unconstitutional.

Congress created a seven-member safety commission to investigate and make recommendations to Congress for new fireworks safety laws, to make further rules for establishing safety and performance standards, and to prosecute violations of these safety standards. The chairman of the commission was appointed by the President. Three members were selected by the Speaker of the House of Representatives, and three members were selected by the President pro tempore of the Senate. An organization with proper standing seeks to enjoin enforcement of the commission's rules. Which of the following presents the strongest constitutional argument that the organization can make against the validity of the commission? Press Enter or Space to submit the answer

The strongest argument is that the commission lacks authority to enforce its standards. The Appointments Clause of the Constitution permits Congress to vest appointments of inferior officers only in the President, the courts, or the heads of departments. Enforcement is an executive act; therefore, Congress cannot appoint members of a commission that exercises enforcement powers. In these facts, the safety commission consists of some members appointed by Congress. This commission therefore violates the Appointments Clause of the Constitution because it has enforcement powers, since it can prosecute violations.

At the trial of the plaintiff's breach of contract action against the defendant, the plaintiff called her accountant as a witness to testify about the difference in gross sales, gross income, and net profit caused by the defendant's failure to supply the promised quantity of ice cream to the plaintiff's ice cream shop. When the plaintiff's attorney asked the accountant to state the gross income figures for the year prior to formation of the contract between the plaintiff and the defendant, the accountant replied that he could not remember the exact amounts. The plaintiff's counsel then handed the accountant a copy of the federal tax return submitted by the plaintiff for that year, and asked him to read it. Counsel then asked, "Now that you have read the tax return, can you remember what the gross income of the plaintiff's ice cream shop was for the relevant period?" The defendant's counsel objects. How should the court rule?

This is admissible testimony. Can legally refresh a witness's memory by showing them almost anything. Can just be used to jog their memory. DIFFERNTIATE FROM PAST RECOLLECTION RECORDED. Past recollection recorded would be if witness read the document and still didn't remember. Could then try to have the document read into evidence if the writing was made by the witness at a time when the facts were known to him.

A man beat his girlfriend and fled. The girlfriend called the police and told them about the beating. She also told them that the man likely fled to his best friend's house. The police obtained a valid arrest warrant for the man and went to the friend's house. They knocked and the friend answered the door. The friend told the police that the man was not there. The police pushed past the friend and began searching for the man. The police did not find the man, but they did find a package of cocaine on a small end table in plain view. The police arrested the friend for possession of cocaine. Prior to trial, the friend moves to suppress the cocaine, claiming that it was unconstitutionally seized. Should the court grant the motion?

Yes, because there was no search warrant. Police MUST have search warrant or exigent circumstances to enter a third-party's home.

The plaintiff was injured when the bus in which she was riding braked too abruptly and threw her into a support stanchion, breaking her hip. She has brought an action against the bus company for damages from personal injuries on theories of respondeat superior and negligent hiring. During the bus company's case in chief, its counsel calls the company's personnel director as a witness and asks him if the driver of the bus had been required to provide proof that he had had no convictions for crimes relating to vehicle use before being hired. The witness answers, "It's been several years since he was hired, but my best recollection is that we did not ask for such proof." Counsel then prepares to question the witness about his statement, made at a deposition taken 18 months before trial, that he had personally requested and received a statement from the driver before he was hired that he (the driver) had no such convictions. May counsel for the bus company pursue this matter in this fashion?

Yes, evidence of a prior inconsistent statement can be used to impeach a witness and can come in as substantive evidence if statement as given under penalty of perjury at a prior trial or proceeding, or deposition.

An employee filed an employment discrimination action against her employer in federal district court. The employee alleges that she has not been promoted because of her gender. She intends to call a co-worker as a witness at trial. The co-worker will testify that a senior manager of the employer told the co-worker that the employee would not be promoted because the employer deemed women to be poor managers. Is the identity of the co-worker subject to discovery by the employer?

Yes, identity of individuals with discoverable information that disclosing party may use to support its claims must be provided. MUST BE DISCLOSED EVEN WITHOUT REQUEST FROM EMPLOYER

Suspecting criminal activity, a police officer acting without a warrant peeked through a small opening in the shutters of an apartment. The officer observed the apartment's tenant and the defendant making methamphetamine. The officer immediately entered the apartment and arrested the tenant and the defendant, and he confiscated the ingredients for the methamphetamine, the tools used for methamphetamine production, and any completed methamphetamine for evidence. The search is later ruled invalid at a suppression hearing. May the defendant now claim that her Fourth Amendment rights have been violated by the seizure of the ingredients, tools, and methamphetamine from the apartment?

Yes, if she was an overnight guest. To raise a Fourth Amendment claim of an unreasonable search or seizure, a person must have a reasonable expectation of privacy with respect to the place searched or the item seized. It is not enough merely that someone has an expectation of privacy in the place searched. The Supreme Court has imposed a standing requirement so that a person can complain about an evidentiary search or seizure only if it violates her own reasonable expectations of privacy. The Court has held that a person has a reasonable expectation of privacy any time (i) she owned or had a right to possession of the place searched, (ii) the place searched was in fact her own home, whether or not she owned or had a right to possession of it, or (iii) she was an overnight guest of the owner of the place searched. Thus, the defendant would have standing to challenge the search of the tenant's apartment if she was an overnight guest of the tenant.

The plaintiff brought a breach of contract suit against the defendant, alleging that the defendant paid for only 25 cases of baseball cards even though the order was for 50. The defendant claimed that only 25 cases were delivered to his store. The plaintiff then introduced a shipping bill from a freight company showing that pickup had been made from the plaintiff on 50 cases. May the defendant now compel the plaintiff to introduce the remainder of the record, showing that the freight company had only 25 cases of baseball cards on its truck?

Yes, they can compel. When a document is shown but pertinent parts are left out, fairness dictates that the opposing side can compel the other side to reveal the other parts of a document that omit pertinent details.

A debtor owed a creditor $5,000, but the debt was barred by the applicable statute of limitations. The debtor agreed to assign to the creditor a $4,000 debt that was owed to him by a third party and was coming due in a week. The debtor called the third party to inform him of the assignment. When the debt became due, the third party refused to pay the creditor. The creditor brings an action to collect the debt against the third party. Will the creditor likely prevail?

Yes. Assignments of credit do not have to be in writing to be enforceable. SOF exception.

A developer contracted with a general contractor to build an office building, and completion of the building was two years late. The developer filed a breach of contract action in federal district court against the general contractor, seeking damages caused by the delay. The general contractor filed a third-party claim against a major subcontractor, claiming that the subcontractor caused any delay and should be liable to the general contractor for anything the general contractor has to pay the developer. The subcontractor believes that the developer interfered with the subcontract and that the developer's interference caused not only the delay but also substantial cost overruns for the subcontractor. May the subcontractor assert a claim in the pending action against the developer seeking payment for the cost overruns?

Yes. THIRD PARTY claims can be brought as long as they arise from same transaction or occurrence, but may also be brought later in separate action. Compulsory claims apply only to ORIGINAL defendants. Here, subcontractor was brought by developer, not plaintiff.

A plaintiff sued an auto manufacturer for negligence after a car accident involving the plaintiff's car that was made by the auto manufacturer. Sixty days after service of the complaint and 40 days after service of the manufacturer's answer that contained no counterclaim, the plaintiff filed a motion seeking to file an amended complaint adding a claim for strict products liability against the auto manufacturer stemming from the same incident. The statute of limitations for strict products liability claims expired one week before the motion was filed. How should the court rule on the plaintiff's motion? A. Grant the motion, because every party is entitled to amend once as a matter of course. B. Grant the motion, because the amended complaint relates back. C. Deny the motion, because, while the motion is timely, the proposed claim is futile because the statute of limitations has run. D. Deny the motion, because it is not timely.

B. Grant the motion, because the amended complaint relates back. The court should grant the motion. Federal Rule of Civil Procedure 15 states that leave of court (to grant motions to amend) is to be "freely given when justice so requires." The rule does not provide any clear date when amendments are no longer permissible, although later amendments obviously would be less fair and less likely to be considered in the interest of justice. Additionally, for statute of limitations purposes, proposed claims may be considered to "relate back" to the date of the original pleading in which the claim was made under Rule 15(c). That is critical here because, although the statute of limitations for the proposed products liability claim had expired at the time the motion was made, the amended complaint asserting a products liability claim relates back to the original filing because that claim stems from the same facts alleged in the original complaint, and the statute of limitations had not expired at the time of the original filing. (A) is incorrect because a plaintiff may amend the complaint once as a matter of course (i.e., without court intervention) not later than 21 days after service of the auto manufacturer's answer.

A consumer filed a breach of contract action against a seller in a state court in State A, seeking $100,000 in damages. The consumer was a citizen of State A. The seller was a State B corporation whose principal place of business was in State A. Five days after being served with the complaint and summons, the seller removed the action to federal district court. Seven months later, the consumer filed a motion to remand the action back to state court. How should the federal court rule on the motion to remand the action to state court?

Allow remand here even though not timely because there was no SMJ. Usually, have to move to remand within 30 days, but that does not apply if the court has no SMJ. Can remand at any time!

A drug dealer was persuaded by his public defender to plead guilty to possession of marijuana, even though the drug dealer believed that the seizure of the marijuana was illegal. He pleaded guilty while protesting that he was innocent. The judge accepted the plea after determining that it was voluntary and intelligent. After the drug dealer was released on bail prior to sentencing, he was advised by a civil attorney that he could sue the arresting police officer to recover damages for an illegal search and seizure of the marijuana. Is the civil attorney correct?

Apparently, guilty plea does not waive right to sue civilly for violation of 4th Amend right

A consumer purchased a new television set from an electronics store. When he got home, he opened the box and found an owner's manual that contained operation instructions, warnings regarding the danger of electricity, and a warranty that stated: "The store expressly warrants that this set shall be free of manufacturing defects for 30 days. If a set is defective, the store's liability shall be limited to the cost of repair or replacement of defective parts. The store "HEREBY DISCLAIMS ANY AND ALL OTHER WARRANTIES, EXPRESS OR IMPLIED, INCLUDING THE WARRANTY OF FITNESS FOR PARTICULAR PURPOSE AND THE WARRANTY OF MERCHANTABILITY."" Five weeks later, after the set was properly installed, the consumer turned on the set, heard a crackling noise, and watched as his television exploded and was destroyed. Under which of the following theories will the consumer most likely recover? Press Enter or Space to submit the answer

Apparetly breach of implied warranty of merchantability. Reason is because no warranties were actually expressly given or disclaimed here because the warranty was in the box. For it to be valid it has to be part of the offer and acceptance process, meaning the buyer would have to have seen these promises before actually purchasing the TV.

A franchisee filed an action against a franchisor in federal district court, asserting claims under federal antitrust statutes. Before filing its answer, the franchisor filed a motion to dismiss the action for failure to join a party under Rule 19 of the Federal Rules of Civil Procedure. Following a hearing, the court denied the motion to dismiss. The franchisor then filed its answer addressing the merits of the claims, and the parties proceeded with discovery. After approximately a year of discovery, the franchisor asserted at trial the defense that the franchisee's complaint failed to state a claim upon which relief can be granted. Did the franchisor timely assert this defense such that the court should address the merits of the defense?

Can bring failure to state a claim motion at any time, not just with 12(b)(6) or in answer.

A member of an out-of-town yacht club agreed to buy a luxury boat from a local yachtsman, a boat which the parties referred to as "the 'Lady' boat" throughout their preliminary negotiations. The yachtsman owned two boats-"Lady Be Good" and "Lady Luck." The yachtsman's intent was to sell "Lady Luck," while the club member intended to buy "Lady Be Good," which the club member knew was once owned by the club member's favorite celebrity. The club member was unaware of the existence of "Lady Luck." The written contract for the yachtsman's "Lady" boat included a sale price of $100,000. When the yachtsman delivered "Lady Luck" to the club member's slip, the club member saw that it was not the boat he had intended to purchase and refused to accept delivery. If the club member sues the yachtsman for specific performance to compel him to deliver "Lady Be Good," and the yachtsman countersues to compel the club member to accept delivery of "Lady Luck," who should prevail?

Club member will prevail. If terms are ambiguous and parties have different interpretations of the meanings, usually neither party will prevail for mutual mistake. However, if one party has reason to know that the other might be mistaken, here because he had two boats named "Lady", the intent of the other party will be enforced. "Two ships Peerless"

Congress enacted a statute imposing an additional 2% national retail sales tax on all goods sold in the United States. The purpose of the tax was to fund unemployment payments during the recession. An association of newspaper and book publishers sued to remove the tax from newspaper and book publications, claiming that the tax placed an unconstitutional burden on the freedom of speech and press. Assuming that the association has standing, will it win the lawsuit?

Congress has plenary power to tax. As long as legislation bears some reasonable relationship to revenue production or if congress has power to regulate taxed activity, it is all good. Here, purpose of funding unemployment payments during recession was valid revenue raising purpose.

A driver was driving his car negligently along a mountain road. He lost control of his car and careened over the side of a cliff. A jogger saw the driver's car go off the cliff and stopped to see if he could help. The jogger started to climb down the cliff to render aid to the driver. In doing so, the jogger slipped and broke his leg. The jogger sued the driver to recover damages for his broken leg. Regarding any defenses the driver might raise, which of the following statements is correct?

Excitement of the accident and speedy response of the rescuer must be considered. Rescuers do not act "in their own peril." A rescuer is a foreseeable plaintiff as long as the rescue is not reckless; hence, the defendant is liable if he negligently puts himself in peril and the plaintiff is injured attempting a rescue. A plaintiff may take extraordinary risks when attempting a rescue without being considered contributorily negligent. The emergency situation is one of the factors taken into account when evaluating the plaintiff's conduct.

A plaintiff sued an auto manufacturer for negligence after a car accident involving the plaintiff's car that was made by the auto manufacturer. Sixty days after service of the complaint and 40 days after service of the manufacturer's answer that contained no counterclaim, the plaintiff filed a motion seeking to file an amended complaint adding a claim for strict products liability against the auto manufacturer stemming from the same incident. The statute of limitations for strict products liability claims expired one week before the motion was filed. How should the court rule on the plaintiff's motion?

Grant the motion, amended complaint relates back. Can relate a claim back if statute of limitations has expired but wouldn't have at the time of the original pleading. Here, claim related back. "Federal Rule of Civil Procedure 15 states that leave of court (to grant motions to amend) is to be "freely given when justice so requires." The rule does not provide any clear date when amendments are no longer permissible, although later amendments obviously would be less fair and less likely to be considered in the interest of justice. Additionally, for statute of limitations purposes, proposed claims may be considered to "relate back" to the date of the original pleading in which the claim was made under Rule 15(c). That is critical here because, although the statute of limitations for the proposed products liability claim had expired at the time the motion was made, the amended complaint asserting a products liability claim relates back to the original filing because that claim stems from the same facts alleged in the original complaint, and the statute of limitations had not expired at the time of the original filing."

A state statute allows for criminal trials by a jury composed of six jurors. Five of the six jurors must concur for a guilty verdict. The defendant is charged with petty larceny, which carries a maximum sentence of one year's imprisonment, plus a fine of $2,500. Before voir dire begins, the defendant objects to both the six-member jury and the fact that only five of six jurors are needed for a conviction. Should the trial judge overrule the objection?

No. No right to 12 jurors, but right to at least 6. In criminal cases, juries must also be unanimous -- here, they were not, and as such this was unconstitutional.

In separate lawsuits, the driver of a car and the passenger both sued the defendant for injuries sustained when the defendant's car struck the driver's car. In the passenger's lawsuit, he testified that he was not paying attention to the other cars on the road at the time of the accident, but offers into evidence the prior deposition testimony of the driver. At that deposition, the driver had testified that she noticed the defendant's car swerving in the lane next to her and suddenly crossing into her lane and colliding with her car. The driver had moved out of state shortly after her lawsuit concluded. No attempt was made to locate the driver or procure her attendance. Should the court admit the deposition testimony over the defendant's objection?

No. While the testimony was made under oath at a prior proceeding, have to show that the witness is now unavailable and there is sufficient similarity of parties and issues with a prior chance to cross-examine. Here, no showing that witness is unavailable, so can't come in. Could if they can show he is unavailable.

A customer selected a new wallet at a local department store that the salesperson said was made of the finest calfskin and was stitched by hand. After having an opportunity to inspect the wallet, the customer bought it and left the store. A few moments later, he took out the wallet to transfer his cash and credit cards into it. On closer inspection, he noticed a small nick in the leather. He immediately went back to the department store and demanded a refund. The salesperson refused. If the customer sues for a refund, who will prevail? Press Enter or Space to submit the answer

Once accepting goods lose the right to inspect unless the breach is substantial and you had a good reason for accepting the goods before inspecting. Here, customer accepted the goods and the breach is too minor to just revoke acceptance.

A tenant remained in possession of the house she was renting after her lease term had expired, prompting the landlord to begin eviction proceedings. While the tenant was still in the house, a heavy snowfall covered the driveway, requiring her to shovel the driveway so she could get her car out of the garage. Shortly after she finished shoveling, the tenant's neighbor used a snowblower to blow all of the snow from his driveway onto the tenant's driveway. Consequently, the tenant had to shovel it again before she could get her car out. Which party may bring a trespass action against the neighbor?

Only the tenant can bring trespass action. Even if they are not legally in possession of the land, if they have construction possession, can sue for trespass. Landlord cannot sue because they do not have possession of the land. They may be able to sue if there is damage to the land that affects the property value but not for trespass.

A homeowner lived next door to a vacant lot owned by another neighbor. From the time the homeowner purchased his own property, he told other people that he owned the vacant lot. The homeowner had an underground dog fence installed under the vacant lot without the neighbor's knowledge. The homeowner also mowed the vacant lot regularly in the summer. When he had landscaping services performed on his own property, the landscapers dug up vegetation beds, which extended three feet into the neighbor's lot. After the statutory period for bringing a trespass action had passed, the homeowner brought an action to quiet title, claiming ownership of the vacant lot. Which of the following additional facts, if true, would be most helpful to the homeowner's case?

Openly telling the actual property owner that you are the owner helps establish the "adverse" prong of COAH for adverse possession.

A pop musician of modest fame offered to sell his first guitar, the one he played as a teenager before he became famous, to a devoted fan for $2,000. The fan paid the musician $50 to keep the offer open for 60 days. Thirty days later, the musician suddenly died. The administrator of his estate found a copy of the agreement and informed the fan that the offer was revoked. Nevertheless, after the value of the musician's paraphernalia skyrocketed due to his death, the fan contacted the administrator to proceed with the sale for $2,000 just before the 60-day period expired. At that time the value of the guitar was estimated to be $50,000. The administrator refused to sell the guitar to the fan for $2,000, so the fan sued the estate for breach of contract. Is the court likely to rule in the fan's favor?

Option contract survives death! This is in contrast to normal contracts that are terminated upon death of one of the parties.

A hiker was passing by an isolated farmhouse one evening when she noticed flames coming from a shed attached to the house, which was dark and apparently unoccupied at the moment. The property on which the house was located was protected by a barbed wire fence, but the hiker happened to have a pair of wire cutters with her, so she cut through the barbed wire fence and entered the property. She put out the fire with a garden hose before it spread to the house. Is the hiker liable for trespass?

Plaintiffs can be deemed to consent to trespass if it is reasonable to belief that a normal person would conclude consent would be given, for example to save person or property during an emergency, as was the case here.

A group of voters filed a civil rights action against a city in federal district court. The complaint alleged that the city's voting procedures violated the United States Constitution and sought an injunction requiring certain changes to the procedure. Two weeks after filing the complaint and before the city filed any response, the group filed a voluntary dismissal of the action because they lost their anticipated funding for the lawsuit. Having secured alternate funding, the group refiled an identical complaint six months later. Two weeks after filing the second complaint and before the city filed a response, the group again filed a voluntary dismissal of the action because the judge to whom the case was assigned had a reputation for denying injunctions. May the group refile the claim and maintain the action?

Plaintiffs get one voluntary dismissal without prejudice. The second voluntary dismissal acts as a final adjudication on the merits and bars it being brought again.=

On August 5, the owner of a hot dog plant and the proprietor of a local ballpark concession stand entered into a written agreement providing, among other things, that if the local team wins the state championship, the plant owner will deliver to the proprietor 500 hot dogs on each of the following days: September 5, 7, and 9. The price was set at 25 cents per hot dog, with payment to be made on September 10 by the proprietor to a creditor of the plant owner. On August 15, the plant owner decided that he wanted to avoid his obligation to deliver the hot dogs. The creditor has not become aware of the agreement between the plant owner and the proprietor. Which of the following is the most accurate statement?

Proprietor would have to agree too. Can't just unilaterally rescind the contract. If both sides agreed before performance then it would be fine.

A homeowner contracted for construction of a custom-built, elevated deck in his backyard. The deck's designer supervised the construction, which was carried out by several employees of a local building company. The homeowner was pleased with the appearance of the deck, but the first time he stepped on it, a support on one side of the deck gave way, causing the homeowner to fall and be injured. The homeowner brought an action joining the building company and the deck's designer as defendants, alleging negligence. In his complaint, he alleged that he does not know which of the defendants is responsible for the damages. Which of the following doctrines would be most helpful against the designer?

Res ipsa. In situations where plaintiff might not known which defendant was negligent but knows there was some type of negligence, and one defendant exercised sufficient control, can use res ipsa.

On April 15, a seller entered into a valid written agreement to sell her home to a buyer for $175,000. The provisions of the agreement provided that closing would be at the buyer's attorney's office on May 15, and that the seller would deliver to the buyer marketable title, free and clear of all encumbrances. On the date of closing, the seller offered to the buyer the deed to the house, but the buyer refused to go ahead with the purchase because his attorney told him that a contractor who had done work on the house had recorded a lis pendens on May 1 against the property regarding a $10,000 contract dispute he had with the seller. The seller indicated that she was unaware of the lien, but that she was willing to go ahead with the sale and set aside funds from the purchase price to cover the contractor's claim until the dispute was resolved. The buyer still refused to proceed, stating that the seller had breached the contract. If the seller brings an action against the buyer for specific performance, what is the probable result?

Seller would prevail here because there is implied agreement in the contract that she could use proceeds from the sale of the home to clear the encumbrance, provided she can guarantee protection for the buyer. Here, she was willing to put money in escrow while she resolved the dispute with the contractor so that would protect the buyer.

An acquaintance asked the defendant to give him a lift downtown because he did not have bus fare. While riding on the defendant's motorcycle, the acquaintance asked to stop at a convenience store to get a bottle of wine, showing the defendant a tire iron in his backpack that he was going to use. The defendant nodded in acknowledgment of what the acquaintance was planning to do. The defendant stopped at the store and waited in the parking lot while the acquaintance went in. He demanded money from the clerk, brandishing the tire iron. The clerk tried to grab a gun under the counter while he was filling a bag with money, and a struggle ensued. The gun discharged, killing the clerk. The defendant heard the gunshot and raced off, but was eventually apprehended. The jurisdiction's criminal code provides that a death caused during the commission of certain felonies, including robbery, is first degree felony murder, for which the death penalty is permitted. The code also permits cumulative penalties for first degree felony murder and for the underlying felony. The defendant was charged and convicted of both robbery and felony murder. After appropriate consideration of all relevant circumstances, the jury imposed the death penalty. On appeal, the defendant challenged both the convictions and the sentence. Assuming that the above facts were properly admitted into evidence, how should the appellate court rule?

THIS IS NOT IN BARBRI OUTLINE OR LECTURE FYI 'The defendant can be found guilty of robbery and felony murder, but the death penalty cannot be imposed. The defendant can be found guilty of robbery as an accomplice. The Supreme Court has held that, under the Eighth Amendment, the death penalty may not be imposed for felony murder where the defendant, as an accomplice, did not take or attempt or intend to take life, or intend that lethal force be employed. [Enmund v. Florida (1982)] Here, because the defendant's involvement in the crime was only to provide transportation, it cannot be said that he participated in such a major way that he acted with reckless indifference to human life; hence, the death penalty cannot constitutionally be imposed against him."

A state study indicated that an inordinately high percentage of homeless in the state were afflicted by alcoholism or addiction to illegal drugs. The legislature therefore decided to levy a special tax, with all proceeds marked for rehabilitative services for the homeless. However, the legislators determined that direct taxes on alcoholic beverages would be resented by the citizenry. Lobbyists from the state's growing wine industry also objected to anything that would retard the industry's development. There were no breweries or distilleries within the state. Thus, a tax was eventually passed requiring newspapers and magazines of general circulation published in the state to be taxed at a rate of 20% on all advertising space sold for beer or distilled spirits promotions. For certain historical reasons, a high proportion of the advertising revenue of a particular small newspaper within the state came from beer and wine ads. The publisher of the small paper filed suit to have the tax declared unconstitutional. A major wholesale beer and liquor distributor located within the state and several out-of-state brewers and distillers who sold and advertised their products in the state also joined in the suit as plaintiffs. If the tax is declared unconstitutional, what is the most likely reason?

Tax infringes on First and Fourteenth Amendment freedom of press here. Press can be taxed if it is a generally applicable business tax, but can't single out the press absent a compelling reason.

The defendant was on trial for the crime of reckless homicide after allegedly running over the victim with his car and killing him. The defendant testified that he was not driving recklessly. Defense counsel wants to call to the stand the defendant's colleague, who regularly shares rides with the defendant to and from work, to testify to his reputation for careful driving and for truthfulness. Should the testimony be admitted? Press Enter or Space to submit the answer

Testimony regarding defendant's' reputation as a careful driver should be admitted! In criminal case, defense can call witness to testify to a defendant's good reputation and opinion for the trait involved in the case at issue, here, that would be as a careful driver. Character for truthfulness is not at issue.

The defendant and the victim, both of whom were drinking, got into a heated exchange of words in a bar. After several minutes, both calmed down and continued drinking. When the victim started to stagger out of the bar after several more hours, the defendant yelled an insult directed at the victim and picked up a chair as if he were going to strike the victim. However, the defendant immediately dropped the chair when the victim turned around. The defendant was charged with assault, and defends on the basis that he was so intoxicated at the time that he did not realize what he was doing. Which of the following would be LEAST helpful to this defense?

That assault is a general intent crime in the jurisdiction. Specific intent requires the requisite intent to actually commit an act, not just the type of recklessness that we see in general intent. Also, voluntary intoxication is only defense to specific intent crimes, not general!

On January 1, a car salesman offered to sell an antique car to a collector for $35,000 cash on delivery. The collector paid the car salesman $100 to hold the offer open for a period of 25 days. On January 4, the collector called the car salesman and left a message on his answering machine, asking him whether he would consider lowering the price to $30,000. The car salesman played back the message the same day but did not reply. On January 9, the collector wrote the car salesman a letter, telling him that he could not pay more than $30,000 for the antique car, and that if the car salesman would not accept that amount, he would not go through with the deal. The car salesman received this letter on January 10 and again did not reply. The car salesman never heard from the collector again. When did the offer that the car salesman made to the collector on January 1 terminate?

The 25th of January. When option contract opened with consideration, even express rejection by the offeree will not terminate the contract unless the offeror detrimentally relies on that rejection.

A wealthy philanthropist owned a mansion built to his exact specifications, featuring a pipe organ built into the wall of the music room. The organ was impressive, with beautiful hand-carved wood scrollwork. The accompanying bench was made from the same wood as the organ and was carved to match the patterns on the organ. The bench was fully movable and could be slid into a niche beside the organ when not in use, although the philanthropist usually left the bench in front of the organ for its matching effect, even when the organ was not being played. The philanthropist died, and his will left all of his personal property to his daughter and all of his real property to a local charity. After the will was admitted to probate, the daughter removed all of the furniture and other movables from the mansion, including the organ bench. The daughter refused the charity's request to return the bench to the mansion. If the charity brings suit against the daughter to replevy the bench, who will prevail?

The charity would be able to keep the bench because it is integrally connected to the organ. An integral part of a fixture that is not actually a fixture itself is known as an accession. Here, the bench was an accession to the fixture organ and, as such, stays with the owner of the land.

The United States Surgeon General was cited for contempt for refusing to answer questions as part of a Senate investigation regarding an issue in the Food and Drug Administration. His contempt citation will be dismissed if he can show which of the following?

The questions must not relate to any matter that the Senate can actually regulate. "His contempt citation will be dismissed if he can show that the questions do not relate to any matter concerning which the Senate may legislate. Congress's power to investigate is limited to matters on which it can legislate. Therefore, if the Surgeon General can demonstrate that the questions concerned matters upon which Congress could not legislate (not an enumerated power under Article I, Section 8), then this contempt citation must be dismissed."

A young woman went to her local shoe shop and selected a pair of shoes. She gave the salesperson cash for the shoes. As the salesperson was putting the shoes into a bag, a robber brandishing a gun entered the store, forced the salesperson to put all of the money in the register into the bag with the shoes, and fled with the bag, the money, and the shoes. After the police had come, the young woman asked the salesperson to get her another pair of shoes. He told the young woman that she would have to pay for them again. The young woman refused. If the young woman sues the shoe shop for another pair of shoes, who will prevail?

The woman because she did not yet have possession of the shoes. For merchants, risk of loss does not pass to buyer until the buyer takes physical possession of the goods.

A victim and his former business partner, the defendant, had a bitter falling out after the victim accused the defendant of embezzling company funds. The defendant threatened to get even. Shortly thereafter, while driving on the expressway, a car swerved suddenly in front of the victim's car. Although the victim applied the brakes immediately, his car failed to stop. To avoid colliding with the car ahead of him, he swerved to the right and smashed into a concrete retaining wall. A passing motorist stopped and came to the aid of the victim. Bleeding profusely from a head wound, and rapidly losing consciousness, the victim said, "I don't think I'm going to make it. My former partner must have tampered with my brakes to get back at me." With that, the victim lapsed into unconsciousness, and has been in a coma and on life support ever since. A personal injury suit has been filed on his behalf by a court-appointed guardian against the defendant. At trial, can the motorist testify as to the statement made by the victim?

This would be applicable because it's a civil or homicide case and the declarant is now unavailable, but the declarant is only speculating. For a dying declaration to come in they must have first-hand knowledge of the incident.

A trial judge presiding over a lawsuit plans to call a witness to the stand and question her. May the judge do so?

Trial judge can call any witness and question a witness, but both parties are then entitled to cross examination.

A college student from State A hit another car driven by a resident from State B when the college student was traveling through State B. The State B driver brought an action in State B state court against the State A college student, who has limited financial resources. The college student filed a motion to dismiss, claiming the State B court lacks personal jurisdiction. What is the best argument to support the college student's motion to dismiss?

Two prongs for PJ - minimum contacts with forum state, pj must not offend notions of fair play and substantial justice. Here, prong one satisfied. Prong two likely satisfied as well, but best argument is that it is unfair to make college student with limited financial means and very limited connection to the state litigate there. A college student from State A hit another car driven by a resident from State B when the college student was traveling through State B. The State B driver brought an action in State B state court against the State A college student, who has limited financial resources. The college student filed a motion to dismiss, claiming the State B court lacks personal jurisdiction. What is the best argument to support the college student's motion to dismiss?

The owner of a pet shop received an e-mail from a professional bird breeder that included the following offer: "Lovebirds! $120 a pair! Delivery in 4-6 weeks. Terms of sale, cash within 30 days of delivery." The shop owner e-mailed the bird breeder on April 26, agreeing to purchase one pair of lovebirds for $120. After looking over the e-mail the next day, the bird breeder sent a reply e-mail to the shop owner indicating that there was a mistake in the initial e-mail; it should have read "$120 per bird." The bird breeder went on to state that she would ship the pair of birds to the shop owner if she would pay the additional $120. In her e-mailed reply, the shop owner authorized the breeder to ship the pair of lovebirds and agreed to pay the additional $120, but she noted that they must be delivered on or before May 21. The bird breeder immediately e-mailed the shop owner agreeing to deliver the birds to her by May 21. However, the birds did not arrive at her shop until June 1, and the shop owner refused to accept them. In an action by the bird breeder against the shop owner for breach of contract, which of the following awards is most likely?

UCC PERFECT TENDER RULE. Because breeder did not get the birds to them in time, no contract. Note that in service contract, courts are more lenient about reasonable time constraints.

Undivided vs. Divided Interest in Land

Undivided: Shared interest in the entirety of the land, eg. If I own half of blackacre undivided, I own half of the interest of the entirety of the property. Divided: Interest in part of the property. Eg. I own blackacre divided, I literally own one-half of the land.

A resident of State B was injured in the Southern District of State A when her car was struck by a large transport truck. At the time of the accident, the truck driver was acting in the course of his employment for a large retail corporation, which owned the truck. The plaintiff intends to file a negligence action in federal district court against both the truck driver and the retail corporation. The truck driver is domiciled in the Central District of State A. The retail corporation is a State C corporation, but has its principal place of business in a city in the District of State D. The corporation operates close to 100 stores in a number of states, including 20 stores in the Southern District of State A. In which federal district court(s) is venue proper?

Venue is proper in the Central District of State A and the Southern District of State A. Federal venue in civil actions is proper in (i) the district where any defendant resides, if all defendants are residents of the state in which the district is located; and (ii) the district in which a substantial part of the events or omissions giving rise to the claim occurred. If there is no district anywhere in the United States that satisfies (i) or (ii), the action may be brought in a judicial district in which any defendant is subject to the court's personal jurisdiction with respect to such action. A corporate defendant is deemed to reside in each district with which it has sufficient contacts to justify personal jurisdiction with respect to the action. The retail corporation thus resides, among other places, in the Southern District of State A, where it is subject to personal jurisdiction with respect to the action based on a specific jurisdiction theory. (Its commission of a tort there would make it subject to specific personal jurisdiction. Operating 20 stores there would be used in the "fairness and reasonableness" assessment.) The truck driver is domiciled in the Central District of State A. Given that both defendants reside in State A, any State A district where a defendant resides is proper, making (B) the correct answer choice.

A homeowner contracted with a local heating company to install two baseboard heaters in an addition to his home for a total cost of $3,500. This figure included the heaters and labor costs for installation. Upon completion of the installation, the heating company sent an invoice to the homeowner for the $3,500. The homeowner did not immediately pay the bill because the heaters were too noisy. The heating company sent a repair worker to the home to service the heaters, but after several attempts to fix the problem, the heaters were still too loud. The homeowner contacted a qualified repairman to find out how to fix the problem and was told it would cost an additional $300 for new blowers and $150 in labor costs to replace the faulty blowers. The homeowner mailed the heating company a copy of the repair estimate and a check for $3,050-the contract price less the cost of new blowers and labor to install them-and wrote prominently on the check "Payment in full for installation of two baseboard heaters." The heating company cashed the check upon receipt. The heating company then sued the homeowner for $450, the difference between the agreed contract price and the amount paid. Is the heating company likely to prevail in its suit seeking the $450 from the homeowner?

When good-faith dispute exists as to the cost of performance, an accord and satisfaction can be accomplished by a good faith tender and acceptance of a check when that check specifically states the check is tendered in full satisfaction of the debt in question. Here, when heating company accepted the check, the accord was satisfied and the homeowner was discharged under the terms of the original contract.

In a drug raid, police in a city searched 25 apartments selected at random in a 300-unit housing project. In a class action approved by the federal court, the 25 tenants sued the city for violation of their constitutional rights. The tenant named as class representative gave notice to all unnamed class members, including another tenant who decided not to opt out. The class action was then certified as a "common question" type. After negotiating with the class representative, the city police agreed to pay each tenant $500 and to conduct no further raids without proper warrants. The representative and the city signed a settlement agreement and a stipulation of dismissal of the class action. The other tenant objects to the amount of damages he is to receive and would rather opt out now and proceed on his own. May the tenant opt out now?

YES. Common question class members are allowed TWO opportunities to opt out.

A landowner conveyed his farm "to my daughter for life, and on her death to her children in equal shares." At the time of the conveyance the daughter had one child, the landowner's grandson. A few years later, the grandson struck a pedestrian with his vehicle. The pedestrian obtained a judgment against the grandson for damages. The jurisdiction has no applicable statute on the matter. Is the grandson's interest in the farm subject to sale to satisfy the pedestrian's judgment?

Yes, creditor can reach it. Any interest that can be transferred freely, here a vested remainder, can be transferred involuntarily to a creditor to satisfy a debt.

As part of a redevelopment plan, a developer purchased a parcel of land and constructed a condominium complex thereon. The deeds to each of the units prohibited any owner from (i) failing to occupy the unit as the owner's principal place of residence for any period, and (ii) leasing the unit at any time for any reason. The restrictions, which were also contained in the contracts that all owners signed, were designed to foster an owner-occupied environment in the redevelopment area. Subsequently, a current owner of a condo unit obtained a job transfer to another state. When he attempted to lease his unit to a friend, the condominium association filed an action for declaratory and injunctive relief. Should the court grant the injunction?

Yes, partial restraint on alienation, such as denial of rent and requirement that the unit be the principal place of residence, is ok in a fee simple. Condos are fee simple!

During the trial of a personal injury case, the plaintiff calls a witness to testify that he saw the defendant spill a slippery substance in the roadway. Following the testimony of the witness, the defendant calls the witness's neighbor, who testifies that the witness has a poor reputation for truthfulness in the community. The plaintiff's attorney then cross-examines the neighbor, asking her, in good faith, if she committed the crime of false pretenses last year. Last year, the neighbor had in fact been charged with and convicted of the crime of false pretenses. The defendant's attorney objects to this question. Should the objection be sustained?

Yes. All that is required to impeach witness is a good faith belief that they may have committed a crime probative of truthfulness. A conviction helps, but is not necessary. For evidence of prior bad acts probative of untruthfulness, cannot introduce extrinsic evidence. Must take witness at their word.

During the defendant's trial for assault and battery, he called to the stand a witness to testify that the victim was the aggressor and the defendant was acting in self-defense. During cross-examination, the prosecution asked the witness if he embezzled funds from his employer five years ago. The witness denied any such activity, and in rebuttal the prosecution seeks to call to the stand the witness's former employer to testify that the witness did in fact embezzle funds during the employment. The defense objects to the testimony. Should the court sustain the objection?

Yes. Cannot introduce prior extrinsic evidence to prove bad acts. If cross-examining about prior act to impeach credibility and defendant denies it, must accept the denial and move on.

In early July, the owner of a toy store entered into a contract with a craftsman for the purchase of 25 hand-carved wooden rocking horses for the holiday sales season. The agreement failed to state a delivery date for the horses. After several months went by without any word from the craftsman, the owner forgot about the agreement. On October 15, the owner ordered a quantity of mass-produced resin-based rocking horses to sell in his store for the upcoming holiday season. One week later, the craftsman arrived at the toy store with a truck carrying the horses and demanded payment, but the toy store owner refused, stating that he already had more than enough rocking horses to sell. The craftsman sued the owner for breach of contract. May the craftsman enforce the contract?

Yes. When no term for contract stated, courts will implement gap fillers under Article 2. For time, it is reasonable in relation to the goods in the contract being delivered. Here, reasonable to expect it would take several months to hand carve 25 horses.

A homeowner returned home from work one day to find a robber in her living room. After a brief physical altercation, the homeowner ran to a bedroom, hid in a closet, and called 911 on her cell phone. Police officers arrived in less than two minutes and were able to apprehend the robber as he tried to run out the front door. Once they made sure he was locked in the police car, one of the officers went to speak with the homeowner about what had happened. She was still crying and shaking when the officer found her, and she said, "Thank you for catching him! He punched me in the head as I was running away!" The robber was charged with robbery and assault. Traumatized, the homeowner left the country and cannot be traced, despite the efforts of the prosecutor. The prosecutor intends to call the officer to testify as to the homeowner's statement. Should the court allow the officer's testimony?

No. This is testimonial evidence -- evidence made to police to to establish or prove past events potentially relevant to a later criminal prosecution. Testimonial hearsay can only come in if the defendant had opportunity to cross-examine the statements made prior. This is inherent in confrontation clause.

On February 3, a property owner mailed an offer to a married couple who had expressed an interest in buying his property at 337 Green Street. The offer asked for $200,000, "terms $60,000 cash, with the balance secured by a first mortgage." The offer reached the couple on February 5. On February 8, the couple replied by e-mail that the offer had been received and was being considered, and added, "We would much prefer a straight cash deal. Would you consider an immediate purchase for $180,000 cash?" On February 10, the property owner responded with a one-word e-mail: "No." After reading the property owner's response, on February 11 the couple e-mailed: "E-mail received. We accept your offer of February 3. Tender the deed c/o our agent, The First National Bank and Trust." The property owner now refuses to sell the property. If the couple sues for specific performance, are they likely to succeed?

Yes. Email Feb 8 was only an inquiry not a counteroffer, which would have eliminated the original offer. Here, original offer stayed open. On February 11, they accepted the still open offer from Feb 3. To form contract need mutual assent - valid offer and acceptance as well as consideration. Here, all prongs satisfied. "A mere inquiry about additional terms or matters is not a counteroffer. The test of whether the reply is a counteroffer or inquiry is whether a reasonable person would believe that the offer was being rejected. Here, the couple's February 8 communication was a mere inquiry, rather than a counteroffer. The couple's statements do not show an outright rejection unless their terms are agreed to; they merely state that the offer was being considered and then ask the property owner to consider their proposal. The February 11 communication is an acceptance."

A wholesaler sued a retailer in a federal court in State A. The retailer timely filed and served a motion to dismiss for lack of subject matter jurisdiction. The court denied this motion. Thereafter, the retailer filed and served his answer. Ten days after serving his answer, the retailer filed an amended answer, raising, for the first time, the defense of lack of personal jurisdiction, which was available when the motion mentioned above was filed. Should the court consider the personal jurisdiction objection? A. No, because that defense has been waived. B. No, because objections to personal jurisdiction may only be made by making a motion to dismiss before filing an answer. C. Yes, because the retailer may serve an amended answer as of right within 21 days after serving his original answer. D. Yes, because the amendment relates back to the original answer, thus preserving his right to raise the objection.

A. No, because that defense has been waived. The court should not consider the retailer's objection because the retailer has waived the defense of lack of personal jurisdiction. A defendant may object to personal jurisdiction in two ways: (i) by raising it in a pre-answer motion to dismiss under Rule 12(b); or (ii) if he has not moved under Rule 12(b), by raising the defense in his answer. Thus, the retailer has waived the defense of lack of personal jurisdiction by making a pre-answer motion to dismiss for lack of subject matter jurisdiction under Rule 12(b) and failing to raise in that motion the defense of lack of personal jurisdiction.

A large insurance company instituted a supplemental benefit plan for its own employees. Under the plan, any employee who had worked for the company for at least 25 years would be permitted to designate a charity to receive, on the employee's retirement, a donation in the employee's name of six months' worth of the employee's salary. The plan gave participating employees an unqualified right to change the beneficiary at any time before payment was made. An employee nearing retirement enrolled in the plan and named his favorite church as the beneficiary of the donation. The church received a letter from the company informing it that the employee had named it beneficiary of his plan and indicating the approximate amount that it would receive upon the employee's retirement in 10 months. The letter did not inform the church of the employee's right to change beneficiaries before that time. Church elders, anticipating the gift, authorized restoration work to the church building, making plans to pay for the work with the funds from the employee's benefit program. Six months later, the employee converted to a different religion and changed the beneficiary of his plan to his new church. When the employee retired, the company paid the benefit to his new church. His old church, which had paid for the restoration work on its completion, demanded payment of the benefit from the company. When payment was refused, the church sued the company. Which party is likely to prevail? A. The church, because the interests of justice require it. B. The church, because its rights as third-party beneficiary had vested when it was informed in writing that it was the beneficiary. C. The company, because the agreement between the employee and the company allowed the employee to change the beneficiary of the benefit plan. D. The company, because it had a duty to pay the employee's new church as the named beneficiary of his plan.

A. The church, because the interests of justice require it. The church will be able to recover against the insurance company because the interests of justice require it. Under the majority view, consideration is not necessary to make an agreement at least partially enforceable where the facts indicate that the promisor should be estopped from not performing. This is stating the concept of promissory estoppel without labeling it as such. Under the Second Restatement, a promise is enforceable to the extent necessary to prevent injustice if the promisor should reasonably expect the promise to induce action or forbearance and such action or forbearance is in fact induced. Here, the insurance company sent a letter to the church informing it that the employee had named the church beneficiary under his employee benefits program. The company did not warn the church that the employee had the right to change his beneficiary and should have reasonably expected that the church would rely on the promise in some way; it is not necessary in charitable contribution cases that the promisor know of a specific expenditure that the recipient made or is going to make. The church did in fact rely on the promise by authorizing and paying for the restoration work. Hence, to prevent injustice, as choice (A) states, the church can recover against the company to the extent of its reliance (the cost of the restoration work).

An electronics company sued a competitor in federal court for patent infringement, alleging that the competitor had copied several aspects of the electronic company's latest smartphone model. The competitor plans to defend the lawsuit but is worried about being forced to reveal confidential research and development information. The parties scheduled a conference under Rule 26(f) of the Federal Rules of Civil Procedure. At the conference, the parties are to discuss whether they need a court order protecting trade secrets and other related confidential information. Is this a proper discussion for a Rule 26(f) conference? A. Yes, because the parties must discuss their discovery plan during a Rule 26(f) conference.Yes, because the parties must discuss their discovery plan during a Rule 26(f) conference. B. Yes, because the sole purpose of a Rule 26(f) conference is to give the parties an opportunity to ask the court for protective orders.Yes, because the sole purpose of a Rule 26(f) conference is to give the parties an opportunity to ask the court for protective orders. C. No, because at a Rule 26(f) conference the parties should discuss only their claims and defenses.No, because at a Rule 26(f) conference the parties should discuss only their claims and defenses. D. No, because a Rule 26(f) conference is used to formulate a plan for trial.No, because a Rule 26(f) conference is used to formulate a plan for trial.

A. Yes, because the parties must discuss their discovery plan during a Rule 26(f) conference.Yes, because the parties must discuss their discovery plan during a Rule 26(f) conference. This is a proper discussion for a Rule 26(f) conference because the parties must develop a discovery plan. At a Rule 26(f) conference, the parties must confer to consider their claims and defenses, the possibility of settlement, initial disclosures, and a discovery plan. The parties then must submit a proposed discovery plan to the court, and the plan must address the timing and form of required disclosures, the subjects on which discovery may be needed, the timing of and limitations on discovery, and relevant orders that may be required of the court.

A boy mowing his lawn noticed a strong vibration from the engine but continued to mow. The engine housing suddenly broke apart and pieces flew off the lawnmower. One piece struck the boy in the head, seriously injuring him. The boy's mother was inside the house and heard yelling from the backyard. She went to the window and saw her son lying on the ground by the lawnmower and a friend of his kneeling over him. She became very upset and fainted. Subsequent investigation showed that a negligent repair by a local mechanic caused the engine housing to shatter. The mother brought a lawsuit against the mechanic, seeking recovery for her son's injury and the emotional distress she suffered. Can the mother recover damages for her emotional distress? A. No, because her son's continuing to mow after noticing the vibration was a superseding cause of the harm. B. No, because the mother was not within the zone of danger from the mechanic's negligence. C. Yes, because the mother was closely related to someone in the zone of danger from the mechanic's negligence. D. Yes, because her son was injured by the mechanic's negligence.

B. No, because the mother was not within the zone of danger from the mechanic's negligence. The mother cannot recover damages for her emotional distress. A duty to avoid negligent infliction of emotional distress may be breached when the defendant creates a foreseeable risk of physical injury to the plaintiff. The mother might try to assert two theories in support of her emotional distress claim, but she is unlikely to prevail on either one. First, she could claim distress flowing from fear for her own safety, but she may prevail only if the defendant's negligence placed her in a zone of danger. Here, because she was safely inside her home and quite distant from the mower when it exploded, she was not in a zone of danger. Alternatively, she could claim distress flowing from her anguish at seeing her son injured. However, for a bystander who is outside the zone of danger from the risk of physical injury but who suffers emotional distress from seeing the defendant negligently injure another, most states allow recovery only if: (i) the plaintiff and the person injured by the defendant are closely related; (ii) the plaintiff was present at the scene of the injury; and (iii) the plaintiff personally observed or perceived the event. Here, while she is related to her son, who was injured by the mechanic's negligence, she was not present at the scene of the injury and did not personally observe or perceive the event. Hence, she cannot recover damages for negligent infliction of emotional distress.

A seller entered into a written contract to sell a tract of land to a buyer. The buyer was to pay $1,500 per month for five years, at which time the seller would deliver a warranty deed. The contract was silent as to the quality of title to be conveyed. After making 12 payments, the buyer discovered that a neighbor had an easement of way over the land, which was not discussed at the time the seller and buyer entered into the contract. The neighbor had not used the easement over the previous year because she had been out of the country. On the basis of the easement, the buyer wishes to cancel the contract. Which party is more likely to prevail? A. The seller, because the neighbor's easement has been extinguished. B. The seller, because the buyer has no basis on which to rescind the contract. Incorrect C. The buyer, because the obligation to convey marketable title is implied. D. The buyer, because the seller has breached the covenant against encumbrances.

B. The seller, because the buyer has no basis on which to rescind the contract. The title was not marketable but the seller had time to cure the defect, here extinguishing the easement. A buyer must give the seller time up until closing to make title marketable. If the title is not marketable at closing, the buyer may rescind.

A defendant was charged with murdering his boss. After obtaining a valid search warrant and executing a valid search of the defendant's office, an officer found a love letter from the defendant's wife to his boss describing their sexual relations. The letter stated, "I can no longer hide my love for you from my husband. I intend to tell him about us and leave him for you." At trial, the officer seeks to testify about the contents of the letter as proof of the defendant's motive for killing his boss. The defense counsel should object on which of the following grounds?

BEST EVIDENCE RULE IF ORIGINAL DOCUMENT IS UNAVAILABLE Here, want officer to testify to the terms of the letter. No reason that a copy of the letter or actual letter itself cannot be shown.

It was common practice in a particular state for a security interest in land to be structured as a deed absolute, which gave a lender absolute title to the borrower's property as security for the loan. The lender would reconvey only on complete payment of the loan by the debtor party, and could dispose of the land immediately without a foreclosure sale on default. A new governor of the state whose campaign platform was built around abolishing the deed absolute mortgage encouraged the legislature to enact a bill that immediately outlawed use of the deed absolute, declaring that all such deeds would be considered mere liens against the secured property. The law applied not only to loans made in the future, but also to the thousands of such loans in existence at the time the legislation was passed. As soon as the governor signed the legislation, lending institutions and individuals who had loaned money secured through deeds absolute challenged the constitutionality of the new law. What is the strongest argument that the challengers can make?

Best argument is that the bill impairs contracts guaranteed under the Contracts Clause. STATE LAWS (not Federal) cannot enact legislation that substantially impairs existing contracts without important interest that is narrowly tailored. Here, proposed legislation would do just that to the contractual rights of the lenders.

In support of a charity fundraising luncheon, three volunteers independently brought to the event a casserole dish made with ground beef. Each of them had prepared her dish in her own kitchen. Another volunteer combined the dishes onto one large serving platter, from which guests at the luncheon served themselves. One of the guests became seriously ill with what the health department later determined to be a bacterial infection from undercooked beef that was in the combined casserole. The guest brought an action against the three volunteers who made the casserole dishes, alleging negligent preparation of the ground beef. Assuming that the guest can establish only the above facts and his injuries, who is likely to prevail in the action? A. The guest, because, under the doctrine of res ipsa loquitur, he has established an inference of negligence. B. The guest, because he can require each of the volunteers to prove that she was not the actual cause of the injury. C. The volunteers, because the guest cannot establish which of the volunteers breached her duty of care. D. The volunteers, because they all were donating their time and food to the event.

C. The volunteers, because the guest cannot establish which of the volunteers breached her duty of care. The volunteers are likely to prevail. The elements of the prima facie case for negligence are (i) a duty owed to the plaintiff, (ii) breach of that duty, (iii) actual and proximate cause, and (iv) damages. Here, the volunteers each owed a duty of care to anyone consuming the food they prepared, including the guest. The facts indicate that at least one of the volunteers breached that duty by improperly preparing or cooking the ground beef. That breach of duty caused the guest to become seriously ill. However, he cannot establish which of the volunteers breached the duty of care and was a factual cause of his injury. Absent additional evidence, the guest will not prevail. (A) is incorrect. Res ipsa loquitur does not apply because more than one person supplied the casserole dish. The res ipsa loquitur doctrine enables a plaintiff to establish breach of duty just from the fact that an injury occurred that would not ordinarily occur unless someone was negligent. However, the plaintiff must establish evidence connecting a particular defendant with the negligence to support a finding of liability against that defendant. When more than one person was in control of the instrumentality that caused the injury, such as here, res ipsa loquitur generally may not be used. The doctrine sometimes has been applied to multiple parties involved in a joint venture, but that does not apply in this case. Each person volunteered independently to make the casserole dish, and each of them worked individually with their own recipes in their own homes, and another person combined the three batches into one casserole dish. (B) is incorrect. The alternative liability or unascertainable cause approach of Summers v. Tice applies when two or more persons have been negligent but it cannot be determined which one caused the plaintiff's injury. The court will shift the burden of proof to each of the negligent defendants to show that his negligence was not a factual cause of the injury. Here, however, there is no evidence that all of the volunteers were negligent; most likely, just one of them was. Hence, the volunteers will not be required to prove that they did not cause the guest's injury.

In a civil action tried to a jury, the defendant objected to the introduction by the plaintiff of certain evidence without the judge's first making a preliminary ruling on the admissibility of the evidence. For which evidence is the defendant's objection not appropriate? A. Opinion testimony regarding the structural integrity of a building by an engineer called by the plaintiff, without a preliminary determination by the judge that the engineer is an expert. B. Hospital records pertaining to the plaintiff offered by the plaintiff, without a preliminary determination by the judge that they were made as a regular activity of the hospital staff. C. Contract negotiations between the plaintiff and a third party, without a preliminary determination by the judge that the third party was the defendant's agent. D. A paramedic's testimony that the plaintiff's wife, before she died, said that the defendant's car went through a red light before hitting her, without a preliminary determination by the judge that she made the statement under a sense of impending death.

C. Contract negotiations between the plaintiff and a third party, without a preliminary determination by the judge that the third party was the defendant's agent. Existence of agency relationships are matters for jury only! Jury is supposed to determine whether offered evidence is relevant; Judge supposed to determine whether offered evidence is competent to be admitted at all. Here, if jury decides that the third party was not the defendant's agent, they would disregard the evidence as irrelevant.

A horse breeder offered to sell a colt to his neighbor and they agreed on a purchase price. The horse breeder subsequently received a letter from the neighbor thanking him for the sale and summarizing their agreement. The letter contained the neighbor's alleged signature. When the horse breeder attempted to set up transfer of the colt, the neighbor denied that she agreed to purchase it. In a breach of contract action against the neighbor, the horse breeder offers into evidence the letter. The horse breeder testifies that he is familiar with the neighbor's handwriting and recognizes the signature on the letter as being hers. Assuming appropriate objection by the neighbor, who claims that she did not sign the letter, how should the trial court rule on the admissibility of the letter? A. Exclude the letter for lack of foundation because lay opinion testimony regarding handwriting identification is not admissible.Exclude the letter for lack of foundation because lay opinion testimony regarding handwriting identification is not admissible. B. Exclude the letter unless its authenticity is established by a preponderance of the evidence.Exclude the letter unless its authenticity is established by a preponderance of the evidence. C. Admit the letter as authentic and instruct the jury accordingly.Admit the letter as authentic and instruct the jury accordingly. D. Admit the letter but instruct the jury that it is up to them to decide whether the letter is authentic.

D. Admit the letter but instruct the jury that it is up to them to decide whether the letter is authentic. The court should admit the letter and instruct the jury that it is up to them to decide whether the letter is authentic. Before a writing may be received in evidence, it must be authenticated by proof showing that the writing is what the proponent claims it is. All that is necessary is proof sufficient to support a jury finding of genuineness. The authenticity of a document is a preliminary fact to be decided by the jury. Here, the horse breeder's testimony that he is familiar with the neighbor's handwriting and that he recognizes the signature on the letter to be that of the neighbor is sufficient to support a jury finding of genuineness. Thus, the letter should be admitted and authenticity should be left to the jury to decide. (A) is wrong because a lay witness who has personal knowledge of the handwriting of the supposed writer may state his opinion as to whether the document is in that person's handwriting. (B) is wrong because authentication of documentary evidence requires only enough evidence to support a jury finding that the matter is what its proponent claims it is. It is not required that the proponent establish its genuineness by a preponderance of the evidence. (C) is wrong because, as noted above, where there is a dispute as to the authenticity of a document, the issue of authenticity is a fact determination for the jury, not the judge, to decide.

A man from a foreign country obtained a doctorate in political science from a state university and applied to teach there. The man was denied employment at the university under a state law requiring all teachers within the state to be United States citizens. Is the state's citizenship requirement constitutional as it applies to the man? A. Yes, because states have the right to set minimal standards for state employees under the Tenth Amendment. B. Yes, because a university political science teacher would exert a great deal of influence over the attitudes of students toward government, the political process, and citizenship. C. No, because the citizenship requirement is not rationally related to a legitimate state interest. D. No, because the citizenship requirement is not necessary to achieve a compelling state interest.

D. No, because the citizenship requirement is not necessary to achieve a compelling state interest. A state generally may not discriminate against aliens absent a compelling state interest, and no compelling interest is served by prohibiting aliens from teaching at a state university. (A) is incorrect. The Tenth Amendment reserves to the states power not granted to the federal government. The Constitution vests the power to regulate aliens in Congress, and thus the states do not have power to control aliens under the Tenth Amendment. (B) is incorrect because it states the standard that the Supreme Court has applied to primary and secondary school teachers. The Supreme Court has upheld state statutes prohibiting aliens from teaching primary or secondary school on the rationale that teachers at the elementary and high school level have a great deal of influence over the attitudes of young students toward government, the political process, and citizenship. It is doubtful that the Court would extend this rationale to university teachers.

A husband and wife were traveling in a car with the wife driving when they were in an accident with a truck. The accident occurred in a jurisdiction that followed the traditional rule as to joint and several liability among tortfeasors. The husband sued the truck driver in federal district court. The truck driver, contending that the wife was an indispensable party, filed a motion to dismiss the action because the husband did not join his wife as a party to the action. How should the court rule on the truck driver's motion?

Deny because a joint tortfeasor is not indispensable party

Owen, the owner of Greenacre, sold it to Alice for $100,000. Alice did not record the deed, and left the country on an extended trip. Owen, seeing an opportunity to make a quick profit, partitioned Greenacre and sold the front half, Frontacre, to Bert in exchange for $50,000. Bert, who knew nothing about Alice's interest in the property, promptly recorded his interest. Two months later, Bert sold Frontacre to Carl in exchange for $55,000. Carl was aware of Alice's interest in Frontacre but recorded his deed to Frontacre anyway. Meanwhile, Owen executed a mortgage on the back half of the property, Backacre, to Bank in the amount of $40,000. Bank knew nothing of Owen's transaction with Alice but neglected to record its mortgage interest. Six months later, Alice returned home and recorded her deed to Greenacre. A statute in the jurisdiction provides: "Any conveyance of an estate in land, other than a lease for less than one year, shall not be valid against any subsequent purchaser for value, without notice, unless the conveyance is recorded." If Alice brings an action to quiet title in Greenacre, how is a court likely to classify her claim?

KNOW THE SHELTER RULE -- CARL HAS SUPERIOR CLAIM TO ALICE BECAUSE HE BOUGHT FROM BERT WHO WAS BFP In addition, the "shelter rule" allows a person who takes from a bona fide purchaser to prevail against any interest that the transferor-bona fide purchaser would have prevailed against, even if the transferee had actual knowledge of the prior unrecorded interest.

The accused was driving his beat-up old car along a narrow road when he was passed by the victim in her new car. The victim's daughter was lying down in the back seat and could not be seen. The accused sped up, drew even with the victim, and repeatedly rammed his car into the side of the victim's car. After several collisions, the victim was forced off the road, rolling down a cliff for several yards. Due to the rolling, both the victim and her daughter were severely injured. The accused was charged with attempted murder of both of them. At his trial, he testifies that he was angry because of the cavalier way the victim passed him in her new car, and that his only intent in smashing into her car was to scratch and dent it so that she would not be so haughty in the future. Assuming that the jury believes this testimony, of whom may the accused be convicted of attempted murder?

NEITHER. ATTEMPTED MURDER REQUIRES SPECIFIC INTENT TO MURDER, EVEN IF ACTUAL MURDER DOES NOT Here, only wanted to scratch and dent the car, not kill them. No specific intent for attempted murder.

A landowner who had owned and operated a small airport notified the electric company that he was discontinuing operations and that it should shut down the electrical current that had supplied his communications equipment. The equipment had been surrounded by a fence and signs warning of high voltage. Because the electric company had maintained a transformer next to the landowner's communications equipment that contained many valuable and reusable parts, it decided to leave the power on to prevent theft until it could schedule removal of the transformer. Three days later, a trespasser who knew that the airport had closed went onto the property looking for something to steal. He could find nothing of value except the transformer. He noticed the signs warning of the high voltage but believed that the power had since been turned off. He scaled the fence with the intent to dismantle the transformer. As soon as he touched the transformer, he was seriously injured by the electric current. If the trespasser asserts a claim against the electric company to recover damages for his injuries, will he prevail?

NO. Cannot use deadly force to prevent property even against trespasser. While trespasser usually would have no rights, they can still recover if using deadly or unreasonable force to protect property. I.e. Spring Gun for door opening.

At the trial of the plaintiff's personal injury action against the defendant, a pedestrian, who was near the accident scene but did not see what happened, testifies that an eyewitness to the accident shouted, "Good Lord! The green car just ran through a red light and hit the red car!" Previous evidence had established that the defendant drove a green car and the plaintiff a red one. The defendant offers to call to the stand the brother of the eyewitness, who will testify that he spoke with the eyewitness the day after the accident, and he said that the light was green when the green car drove through the intersection. The eyewitness had moved to a foreign country prior to trial. Should this evidence be admitted over the plaintiff's objection?

Proper for purposes of impeachment, not substantive. Prior inconsistent statements of a witness can always come in to impeach a witness, but can only come in as substantive when the witness testified under oath at a prior trial or deposition.

A landowner's will left his ranch to a rancher, his heirs, and assigns, so long as the property was used exclusively for ranch purposes, then to the landowner's grandson. The remainder of the landowner's property passed through the residuary clause of his will to the grandson. Seven years after the landowner's death, the rancher began strip mining operations on the ranch. The grandson brought an action to quiet title to the ranch against the rancher, and the rancher counterclaimed on the same theory. Who should prevail?

RAP prevents the executory interest. Court would then read the conveyance without the language of the executory gift, which gives reverter in grantor. As such, intent was to go to grandson and he gets it.

A large-scale bakery in the South entered into a written contract with a commercial apple orchard in the upper Midwest to purchase 200 bushels of apples at a cost of $8 per bushel. The contract provided that the apple orchard would deliver the apples "F.O.B. Louisville Railroad Depot," where the apples would be loaded onto a train headed south. The orchard assigned all of its rights under the contract to a large produce distributor which, in turn, hired a trucking company to deliver the apples to Louisville. En route to Louisville, the truck skidded off the road due to inclement weather and overturned, and the apples were destroyed. The bakery brought suit against the apple orchard for breach of contract. What will be the probable outcome of the litigation?

When FOB designation applies, that designee takes on the risk of loss. If loss occurs before delivery to the FOB location i.e. FOB Louisville, the designee will way the AMOUNT NECCESSARY TO REPLACE DESTROYED GOODS, not necessarily the contract price.

A defendant was involved in an accident in which her car struck the rear end of the car driven by the plaintiff.The police issued tickets to the defendant, charging her with reckless driving and speeding. When the defendant's case came before the traffic court, her attorney entered into a plea bargain with the prosecutor.Under the plea bargain, the defendant agreed to plead guilty to speeding and to pay a fine of $100, and the prosecution agreed to drop the reckless driving charge. Accordingly, the defendant pleaded guilty and the court fined her $100. In the later civil suit, where the plaintiff is seeking damages from the defendant for personal injuries, is the guilty plea before the traffic court admissible?

Yes as stamenet by an opposing party, not statement against interest! Statements against opposing parties are often called admissions, which this would be. (B) is wrong because there is no indication that the defendant is unavailable. Statements of a person, now unavailable as a witness, against that person's pecuniary, proprietary, or penal interest when made are admissible under the statement against interest exception to the hearsay rule. [Fed. R. Evid. 804(b)(3)] A declarant is unavailable if: (i) she is exempt from testifying because the court rules that a privilege applies, (ii) she refuses to testify despite a court order to do so, (iii) she testifies to not remembering the subject matter, (iv) she is dead or ill and unable to testify, or (v) she is absent and the statement's proponent has been unable to procure her attendance or testimony by process or other reasonable means. [Fed. R. Evid. 804(a)(1) - (5)] Since the defendant apparently is available as a witness in the suit, the statement against interest exception is inapplicable.

A retailer of personal watercraft agreed to sell to a buyer a speedboat for $10,000. The written contract specified delivery within 30 days and a down payment of $2,000, but did not contain a liquidated damages clause. Two weeks after making the down payment, the buyer told the retailer that he could not afford to go through with the purchase, and asked for his down payment back. The retailer, which could get as many of that model of speedboat as it required from the manufacturer for a wholesale price of $7,000, put the boat back in its inventory. The retailer then sold it to someone else for $10,500. The buyer sues the retailer to get back his deposit; the retailer counterclaims for damages. Excluding incidental costs, which of the following amounts represents the most likely recovery?

$1000. Retailer paid $7,000, buyer put down $2,000, and the selling price was $10,000. Retailer's expected profit here was $3,000 and, because buyer put down $2,000 already, the leftover $1,000 would get them to $3,00 and as such would be the damages. LOSS PROFITS DOCTRINE -- APPLIES TO VOLUME SELLERS -- Seller is entitled to the value of their lost profits minus any type of down payment already made.

A landowner owned two heavily wooded adjoining parcels of land containing a number of lakes. She conveyed the eastern parcel, which contained a hunting resort, consisting of a number of rental cabins, to a neighbor. The deed transferring the parcel also granted to "the neighbor, his heirs and assigns, and to invited guests of the resort all hunting rights and use of the woods on the western parcel for the benefit of the resort." Subsequently, the neighbor assigned his hunting rights to a hunter. When the landowner discovered the hunter hunting on her land, she brought an appropriate action to declare his rights void. If the court rules for the landowner, it will be because the neighbor's right to hunt on the western parcel is:

A profit appurtenant. Here, profit appurtenant was the hunting rights in the dominate estate that the neighbor owned. CANNOT JUST GIVE THOSE RIGHTS AWAY WITHOUT ALSO GIVING THE LAND AWAY! A profit is a nonpossessory interest in land that entitles the holder of the profit to enter on the servient tenement and take something off of the land (e.g., minerals, timber, oil, or game). Like an easement, a profit may be appurtenant or in gross. If the profit exists to serve a dominant estate, the profit is appurtenant and can only be transferred along with the dominant estate. Conversely, if the profit does not exist to serve a dominant estate, it is a profit in gross and may be transferred separate and apart from the dominant estate. Here, the neighbor has a profit with respect to the game on the western parcel. Because the profit is "for the benefit of the resort" on the eastern parcel, it is appurtenant rather than in gross because it serves the dominant estate (the eastern parcel). Thus, the neighbor's assignment of the profit to the hunter is void.

A plaintiff filed a civil action based on negligence against a defendant in federal district court, alleging that the defendant negligently ran a red light at an intersection and collided with the plaintiff's vehicle, causing the plaintiff's injuries. A week after the close of discovery, the plaintiff filed a motion for summary judgment on the issue of whether the defendant was negligent. With the motion, the plaintiff filed (i) his own sworn affidavit, which stated that the traffic signal was green as he entered the intersection; (ii) an affidavit of a witness who was driving the car behind him, which stated that the witness saw the entire incident and that the plaintiff's traffic signal was green as he approached and entered the intersection; and (iii) an affidavit of another witness, which stated that she saw the entire incident and that the defendant's signal had been red for several seconds before the defendant entered the intersection and was still red when the defendant entered the intersection. In response to the motion, the defendant filed his own affidavit which stated that he does not recall seeing the traffic signal before entering the intersection, but believes that it was not red. He also filed the affidavit of a pedestrian on the scene. The pedestrian stated that she did not see the traffic signal prior to the accident, but that another passerby told her that the traffic signal was yellow for both drivers. How should the court rule on the plaintiff's motion? A. Grant the motion, because the plaintiff has supported his motion with substantial evidence and the defendant has failed to produce admissible evidence that contradicts the plaintiff's evidence. B. Grant the motion, because the defendant's conduct is still negligent even if the defendant's traffic signal was yellow. C. Deny the motion, because the defendant has presented evidence that contradicts the evidence presented by the plaintiff. D. Deny the motion, because a party may not obtain summary judgment on an issue on which that party has the burden of proof.

A. Grant the motion, because the plaintiff has supported his motion with substantial evidence and the defendant has failed to produce admissible evidence that contradicts the plaintiff's evidence. The court should grant the motion. Summary judgment may be granted if, from the pleadings, affidavits, and discovery materials, it appears that there is no genuine dispute of material fact and the moving party is entitled to judgment as a matter of law. The court may not decide disputed fact issues on a motion for summary judgment; if there is a genuinely disputed material fact (meaning a dispute backed by evidence on both sides of the issue), the case must go to trial. Here, the plaintiff's evidence that the light was green when he entered the intersection has not been contradicted by admissible evidence. The defendant's affidavit essentially does not dispute the color of the light, as the defendant cannot remember what color it was. The pedestrian's testimony constitutes inadmissible hearsay. Thus, there is no material fact in dispute, and the motion should be granted.

Using his cellphone, a witness recorded a speeding driver hitting a pedestrian. The witness sold the recording to the driver. The driver then gave the recording to his attorney. After the pedestrian filed suit against the driver, the pedestrian sent a discovery request to the driver requesting that he produce "all items that show or describe the accident." Which of the following best describes whether the driver must provide the video, or a copy of it? A. The driver must provide the video because it is relevant to the pedestrian's claim. B. The driver need not provide the video because, although relevant to the pedestrian's claim, it would be damaging to the driver's defense. C. The driver need not provide the video because he paid money for it, entitling him to possess it.T D. The driver need not provide the video unless the pedestrian can show substantial need and the inability to obtain the equivalent without undue hardship, because the video constitutes work product.

A. The driver must provide the video because it is relevant to the pedestrian's claim. The driver must provide the video. Parties are entitled to discovery that fits under Rule 26(b) (1), which includes "any nonprivileged matter that is relevant to any party's claim or defense." In other words, even if the matter is only relevant to the opposing party, it would still be covered. Additionally, Rule 34 requires a party to produce relevant physical material, including electronically stored information, such as the recording here. There is no exception to relevance for matter that is damaging to a party, so (B) is incorrect. (Being damaging to the defendant's defense might be reason not to disclose it as an initial disclosure because the defendant would not use the recording to support his defense, but the recording would have to be disclosed on a proper request.)

Congress enacted a statute that purported to ban all discrimination against African-Americans in any commercial transaction taking place within the United States. Would the statute most likely be held constitutional? A. Yes, under Thirteenth Amendment provisions barring badges or incidents of slavery. B. Yes, because the federal government has an important interest in furthering the equal protection provisions of the Fourteenth Amendment. C. No, because Congress's powers under the Commerce Clause do not extend so far as the statute would require. D. No, because commercial transactions are not among the privileges or immunities of national citizenship.

A. Yes, under Thirteenth Amendment provisions barring badges or incidents of slavery. The statute is constitutional as a legitimate exercise of congressional enforcement powers under the Enabling Clause of the Thirteenth Amendment. The Thirteenth Amendment prohibits slavery. The Enabling Clause of the amendment has been held to confer on Congress the authority to proscribe almost any private racially discriminatory action that can be characterized as a badge or incident of slavery. Because the statute at issue bans all discrimination against African-Americans in commercial transactions, it necessarily reaches private conduct. Such congressional action is constitutionally permissible pursuant to the Thirteenth Amendment. (B) is incorrect. Application of the Fourteenth Amendment has been limited to cases involving state action. [See United States v. Morrison (2000)] The statute here reaches private action, and so the Thirteenth Amendment is the correct source for the law, since that amendment addresses private action.

A landowner included in his will a provision giving "all of my property, both real and personal, wherever situated, to my widow for life, and after her death to any of our children who may survive her." What is the gift to the children? A. contingent remainder. B. A vested remainder. C. A shifting executory interest. D. Void, as violating the Rule Against Perpetuities.

A. contingent remainder. The children have a contingent remainder. A remainder is a future interest created in a transferee that is capable of taking in present possession on the natural termination of the preceding estate created in the same disposition. Note that, as a rule of thumb, remainders always follow life estates. A remainder will be classified as contingent if its taking is subject to a condition precedent, or it is created in favor of unborn or unascertained persons. Here, the interest in the children follows a life estate and is a remainder because it is capable of taking in possession on the natural termination of the preceding estate. It is subject to the condition precedent of surviving the landowner's widow and, additionally, is in favor of unascertained persons (the children who survive the landowner's widow will not be ascertained until her death). Thus, the interest is a contingent remainder. (B) is incorrect because a vested remainder can be created in and held only by ascertained persons in being, and cannot be subject to a condition precedent. As discussed above, the will provision clearly does not satisfy these requirements because the takers are not ascertained and their interest is subject to a condition of survival.

An infant was injured in an automobile accident when the vehicle, driven by the infant's mother, left the roadway and rolled over down an embankment. At the time of the accident, the infant was buckled into an infant carrier car seat. The carrier was designed to snap into a base that was secured in the back seat by the rear center seat belt. Prior to driving, the mother had snapped the car carrier onto the base and pulled up on the car carrier's handle to ensure that the carrier was indeed secured in the base. When the rollover occurred, however, the carrier came loose from the base and was thrown about the inside of the vehicle, causing injuries to the infant's neck and face. The mother brought a products liability action on behalf of the child against the manufacturer of the car carrier, alleging that the manufacturer was negligent in the design of the base and seat combination. If the mother establishes at trial that the force of the rollover was enough to knock the seat loose, and that a reasonable, economically feasible alternative design existed, which of the following, if true, would be most helpful to the manufacturer's defense? A. The mother violated a statute by traveling too fast for conditions, which caused the rollover accident. B. No one had reported a car carrier coming loose in a rollover prior to this accident. C. The car seat conformed with federal labeling requirements. D. The retailer who sold the car seat was negligent in failing to notice the defect.

B. No one had reported a car carrier coming loose in a rollover prior to this accident. The most helpful fact is that no one had reported this type of problem previously. The mother is alleging that the manufacturer's negligence led to the supplying of a defective product. To establish this, the plaintiff must show that those designing the product knew or should have known of enough facts to put a reasonable manufacturer on notice about the dangers of marketing the product as designed. Negligent design is not shown, however, if the danger of the product becomes apparent only after the product reaches the public. Hence, the absence of any previous complaints about this problem would be most helpful to the manufacturer. (C) is less helpful than (B). Although compliance with government safety standards, such as labeling, is evidence that the product is not defective, it is not conclusive evidence, and federal labeling requirements do not preempt state products liability laws on defective warnings.

A statute in the jurisdiction, which was enacted with the express purpose of preventing public employees from taking advantage of the status of undocumented immigrants, made it a felony to accept money or other benefits in exchange for issuing a state identification card. During an undercover investigation, an undocumented immigrant was recorded offering $500 to a clerk in exchange for issuance of a card. The clerk agreed to the deal and later that day exchanged the card for the money, after which both parties were arrested. In a jurisdiction following the common law approach to conspiracy, which of the following statements is correct? A. The clerk can be convicted of violating the statute and conspiracy to violate the statute, and the undocumented immigrant can be convicted of no crime. B. The clerk can be convicted only of violating the statute, and the undocumented immigrant can be convicted of no crime. C. The clerk can be convicted only of violating the statute, and the undocumented immigrant can be convicted as an accomplice to violation of the statute. D. The clerk can be convicted of violating the statute and conspiracy to violate the statute, and the undocumented immigrant can be convicted of conspiracy to violate the statute.

B. The clerk can be convicted only of violating the statute, and the undocumented immigrant can be convicted of no crime. The undocumented immigrant cannot be convicted of a crime under the statute because it was enacted for his protection, and the clerk cannot be convicted of conspiracy under the statute because the undocumented immigrant, who would otherwise be liable as an accomplice, is not subject to conviction because of a legislative intent to exempt him. If a statute is intended to protect members of a limited class from exploitation or overbearing, members of that class are presumed to have been intended to be immune from liability, even if they participate in the crime in a manner that would otherwise make them liable. Thus, the undocumented immigrant would not be liable as an accomplice under the statute, making (C) incorrect. The clerk clearly can be convicted for the substantive offense, but he cannot be convicted of conspiracy. One of the implications of the common law requirement that there be at least two guilty parties in a conspiracy arises when the crime involves members of a class protected by the statute. If members of a conspiracy agree to commit an act that violates a statute designed to protect persons within a given class, a person within that class not only cannot be guilty of the crime itself, as discussed above, but also cannot be guilty of a conspiracy to commit the crime.

To satisfy a debt owed to a creditor, a son executed and delivered to the creditor a warranty deed to a large tract of undeveloped land. The creditor promptly recorded the deed. Shortly thereafter, she built a house on the property and has lived there ever since. The son never actually owned the land. It belonged to his father, but the father had promised to leave the property to the son. Later, the father died and his will devised the property to the son. Pressed for money, the son then sold the land to an investor by warranty deed, which the investor promptly recorded. Although the investor paid full value for the property, he purchased it strictly for investment and never visited the site. He therefore did not realize that the creditor was living there, and knew nothing of the son's earlier deed to the creditor. The jurisdiction in which the land is located has the following statute: "A conveyance of an estate in land (other than a lease for less than one year) shall not be valid against any subsequent purchaser for value without notice thereof unless the conveyance is recorded." Which of the following is the most likely outcome of a quiet title action brought by the creditor against the investor? A. The creditor prevails, because the son had no title to convey to the investor. B. The creditor prevails, because the investor was not a purchaser for value without notice of the creditor's interest.The creditor prevails, because the investor was not a purchaser for value without notice of the creditor's interest. C. The investor prevails, because under the doctrine of estoppel by deed, title inures to the benefit of the original grantee only as against the grantor.The investor prevails, because under the doctrine of estoppel by deed, title inures to the benefit of the original grantee only as against the grantor. Incorrect D. The investor prevails, because under the recording acts, the deed from the son to the creditor was not in the chain of title and hence did not constitute notice to the investor.

B. The creditor prevails, because the investor was not a purchaser for value without notice of the creditor's interest.The creditor prevails, because the investor was not a purchaser for value without notice of the creditor's interest. The fact that the creditor built a home and was living on the property gave the investor constructive notice of her interest. A title search is not complete without an examination of possession. If the possession is unexplained by the record, the subsequent purchaser is charged with knowledge of whatever an inspection of the property would have disclosed and anything that would have been disclosed by inquiring of the possessor. Therefore, the investor is charged with knowledge of the creditor's possession and with what the creditor would have told him about her possession; i.e., that the property was conveyed to her by the son prior to his conveyance to the investor. Consequently, the investor does not qualify as a bona fide purchaser, and (C) is an incorrect choice. (

A dog owner lived next door to a day care center. Because he had a large yard and there were no applicable zoning restrictions, he installed a kennel and began training attack dogs to sell to businesses. As soon as he opened the business and posted signs in front advertising the exceptional ferocity of the dogs, some parents who had children enrolled in the day care center became alarmed at the prospect of the dogs right next to the yard where the children played, especially because the children could see and hear the dogs being taught to attack people. Within a few months of the dogs' arrival next door, the owner of the day care lost 10% of her enrollment. If the day care owner brings a nuisance action against the dog owner, what will be the most critical factual issue that the trier of fact must resolve to determine who should prevail? A. Whether the day care owner suffered other damages in addition to her economic losses. B. Whether the day care owner's use of her property makes her business abnormally sensitive to the presence of the dogs. C. Whether the dog owner conducted his business with reasonable care. D. Whether the dog owner was apprised of the day care owner's concerns and did nothing to alleviate them.

B. Whether the day care owner's use of her property makes her business abnormally sensitive to the presence of the dogs. The determining factor for the day care owner in prevailing will be whether her use of the property is abnormally sensitive to the presence of the dogs. Nuisance is an invasion of private property rights by conduct that is either intentional, negligent, or subject to strict liability. Strict liability will be the basis for a nuisance action (sometimes called an "absolute" nuisance or a "nuisance per se") when wild animals or abnormally dangerous domestic animals are involved, or when defendant is engaged in an abnormally dangerous activity. Thus, dogs known by their owner to be vicious may create a private nuisance when they interfere with the use and enjoyment of the land next door, and the owner may be subject to strict liability because of his knowledge of the dogs' dangerous propensities. [See Restatement (Second) of Torts §822, comment j] For the presence of the dogs to be an actionable nuisance, however, they must result in a substantial interference with the day care owner's use of her land. The interference will not be characterized as substantial if it is merely the result of plaintiff's specialized use of her own property.

A nephew asked his uncle, who like him was a farmer, to guarantee a loan to buy a new tractor. The local bank had already refused to extend credit to the nephew alone to buy the tractor. The uncle was inclined to refuse, but then decided that he could benefit from his own use of the tractor, so he told his nephew that he would guarantee the loan if he could use the new tractor without cost for 10 days during his harvest season. The nephew agreed to his uncle's proposal. The uncle went to the bank and told the loan officer that he was willing to guarantee the proposed loan to his nephew. This prompted the loan officer to agree to extend the requested credit to the nephew. Although the loan officer did not make the uncle sign any papers, the uncle provided consideration and the bank issued the nephew a loan commitment statement. That evening, the uncle had a change of heart. The next day, he telephoned the loan officer and told him to forget about his guaranteeing any loan to his nephew. Despite the uncle's phone call, the loan officer did not stop the check from being issued, and the nephew received the money to purchase the tractor. He drove the tractor over to the uncle's farm and delivered it for the uncle's 10-day use, as promised. The uncle told his nephew that he did not want to use the tractor and that he was not guaranteeing his loan. Within six months, it became clear that the nephew could not make good on the loan. If the bank sues the uncle for the unpaid portion of the loan, who will likely win?

Bank will prevail because the Uncle's main purpose for guaranteeing the loan was to serve his own interest. SELF INTEREST TAKES AGREEMNTS LIKE THIS OUT OF THE STATUTE OF FRAUDS "Under the Statute of Frauds, certain agreements must be evidenced by a writing that contains: (i) the identity of the party sought to be charged; (ii) identification of the contract's subject matter; (iii) terms and conditions of the agreement; (iv) recital of consideration; and (v) signature of the party to be charged, or of his agent. One type of agreement that is covered by the Statute of Frauds is a promise to answer for the debt or default of another where the promise is collateral rather than primary. However, where the main purpose or leading object of the promisor is to secure an advantage or pecuniary benefit for himself, the contract is not within the Statute of Frauds, even if the effect is still to pay the debt of another.

A storeowner properly filed a complaint for breach of contract against a food distributor in federal district court. After the food distributor timely answered the complaint, the parties proceeded through discovery, which lasted over 14 months and cost the parties over $200,000 in attorneys' fees and related costs. At the final pretrial conference, the presiding judge indicated that he did not think much of the merits of the storeowner's claims. As a result, the storeowner wants to dismiss this case and refile in a different federal court to get a more sympathetic judge. How may the storeowner try to achieve this goal?

Court has DISCRETION to voluntarily dismiss case once without prejudice. However, if significant time and money spent, probably won't allow it.

A debtor owed a creditor $1,200 on a promissory note that was due on August 1. After the debtor told the creditor that he might not be able to pay the note on its due date, the creditor agreed to extinguish the debt if the debtor, who was the manager of a discount electronics store, bought a new entertainment system that sold for $1,200 and had it delivered to the creditor's home by August 15. Because the debtor would have to pay only $600 for the system due to his manager's discount, he agreed and the parties signed a written contract on July 26. Is the new agreement between the debtor and the creditor legally enforceable?

By changing the obligation of the debtor from paying back $1200 to buying an entertainment system, the accord was valid. Accord is valid when one party to an existing contract agrees to accept, in lieu of the performance he is supposed to receive, some type of different performance. Here, that was getting the speaker system instead of straight payment.

Federal narcotics officers suspected the defendant of growing marijuana in his greenhouse, which was connected to his house. The narcotics officers learned from an anonymous informant that the semi-opaque panes of glass on the greenhouse were being replaced during the night with a newer type of glass that let in more light without an increase in visibility. Without a warrant, the officers flew over the defendant's greenhouse in a helicopter that night. One of the officers focused on the greenhouse with a pair of "night vision" thermal imaging binoculars supplied by the Department of Defense and not available to the general public. He determined that marijuana was being grown. The officers then went to a magistrate, swore out a warrant, and arrested the defendant. If the defendant moves to suppress any evidence gathered by virtue of the flyover, how should the court rule on the motion? A. Deny it, because the police may conduct flyovers to gather evidence B. Deny it, because the defendant did not live in the greenhouse. C. Grant it, because the "night-vision" binoculars were not available to the general public. D. Grant it, because the flyover was prompted by a tip from an anonymous informant.

C. The use of thermal imaging binoculars to observe the marijuana where it could not be observed by simply using the naked eye likely renders the search invalid. To be able to assert a Fourth Amendment right, a person must have a reasonable expectation of privacy with respect to the place searched or the item seized. There is no such expectation of privacy in objects or places held out to the public or that may be viewed from a public vantage point. Thus, the police may fly over an area to observe it with the naked eye, and even a low flyover by a helicopter to view inside a partially covered building is permissible. This is true even if the area is within the curtilage. However, the police may not use technologically enhanced methods that are not available to the public to search areas (at least as to areas within the curtilage). In the instant case, the police have flown over the defendant's greenhouse at night and used a means of enhancing their vision that is not available to the general public. This enabled them to see what could not have been observed with the naked eye, which likely constitutes an impermissible search. [See Kyllo v. United States (2001)] (A) is incorrect because it is too broad a statement. The police generally may look into any area, even an area within the home or curtilage, by means of a flyover, so long as they do so from a place the public has access to; however, the police may not use technological enhancements that are not available to the public.

The defendant was fired from his sales job while calling on customers in another city. He failed to return the company car that he was using for his sales visits; instead, he sold the car to a "chop shop" for cash. As to the car, what crime has the defendant committed? A. Larceny B. Larceny by trick. C. Embezzlement D. Theft by false pretenses.

C. Embezzlement Embezzlement because he was in rightful possession of the property when selling the car. Embezzlement is the fraudulent conversion of the property of another by a person in lawful possession of it. In the instant case, the car belonged to the company for which the defendant worked, but the company probably gave the defendant lawful possession of it so that he could make sales calls.

A foreign student who had entered the United States on a student visa four years ago was notified by federal immigration authorities that he was subject to being deported because his visa had expired. Federal law provided that an alien who is subject to being deported has the right to appear before an administrative officer appointed by the Attorney General's office for a hearing on whether he should be deported. This officer, appointed by the executive branch of the government, has the right under law to make a final order concerning whether the alien should be deported. After a hearing, the administrative officer entered an order allowing the student to remain in the United States as a permanent resident. However, a congressional rule permitted the House of Representatives, by resolution, to deport "undesirable aliens." After the administrative judge entered his order, the House passed a resolution that the student should be deported. The student petitioned the federal court to declare the legislative resolution invalid. Should the court find the resolution to be valid? A. Yes, because Congress has plenary powers with regard to aliens and naturalization. B. Yes, because aliens are not "citizens" within the meaning of the Fourteenth Amendment. C. No, because the federal law removed congressional power with regard to aliens in this circumstance, and the resolution of the House violates the separation of powers doctrine. D. No, because the student was denied due process when he was not given a hearing before the House of Representatives.

C. No, because the federal law removed congressional power with regard to aliens in this circumstance, and the resolution of the House violates the separation of powers doctrine. The court should find the resolution invalid. While Congress has broad power to delegate, the separation of powers doctrine forbids Congress from trying to control the exercise of the power delegated in various ways, such as by overturning an executive agency action without bicameralism (i.e., passage by both houses of Congress). By enacting the federal law allowing the administrative law judge to enter a final order with regard to aliens, Congress has given up any control it may have had previously in these situations. The resolution by the House here is an unconstitutional legislative veto that violates the separation of powers doctrine. (A) is incorrect because, while Congress does have plenary power over aliens with regard to immigration and naturalization, here it has given up control over this area by enacting a law allowing an administrative officer appointed by the executive branch to make a final order concerning whether an alien should be deported.

A hardware store ordered 200 cans of wood stain in various shades. The written contract between the store and manufacturer provided that 100 cans of stain would be delivered on April 30, and the remaining 100 cans would be delivered on June 30. Payment would be due at the time of each delivery. The first shipment arrived on April 30. Sales of the stain were brisk, but 25 customers almost immediately returned their stain, complaining that it was not the color indicated on the can. The store owner called the manufacturer and informed it of the problem. The manufacturer truthfully told the owner that they had had a small problem with their labeling machine and a few cans in the store owner's lot must have been mislabeled before they caught the problem. The manufacturer offered to replace all 100 cans from the original order. The store owner refused the offer and told the manufacturer not to deliver the second lot, because he could no longer trust the manufacturer. The owner was very sensitive to the hardware store's good reputation, which he felt was harmed by this incident. If the manufacturer brings a claim of breach regarding the second shipment which was due on June 30, how will the court likely rule? A. The buyer had the right to cancel the second shipment, because of legitimate fears that it would contain the same defects as the first shipment. B. The buyer had the right to cancel the second shipment, because the first delivery was defective. C. The buyer did not have the right to cancel the second shipment, because the defects in the first shipment did not substantially impair the value of the entire contract. D. The buyer did not have the right to cancel the second shipment, because he failed to make a demand upon the manufacturer for adequate assurances that the second shipment would be free of defects.

C. The buyer did not have the right to cancel the second shipment, because the defects in the first shipment did not substantially impair the value of the entire contract. This is an installment contract (deliveries at different times and payment after each delivery) so the contract can only be cancelled if the breach is substantial. Here, breach was not substantial and the manufacturer offered to cure. As such, the buyer could not cancel the deal.

A cyclist was riding on a sidewalk when someone in a parked car suddenly opened the door of the car into her path. She swerved to avoid the car door and rode onto a landowner's property, damaging some plastic lawn ornaments of waterfowl placed in his front yard. In a suit by the landowner against the cyclist for the damage to his lawn ornaments, what is the likely result? A. The cyclist is liable because she had no privilege to enter onto the landowner's property. B. Whether the cyclist is liable depends on whether she was exercising due care. C. The cyclist is liable for the damage to the lawn ornaments even though her entry was privileged. D. The cyclist is not liable for the damage to the lawn ornaments because her entry was privileged.

C. The cyclist is liable for the damage to the lawn ornaments even though her entry was privileged. The cyclist is liable for damage to the lawn ornaments even though she had a privilege to enter the landowner's yard. Pursuant to the privilege of necessity, a person may interfere with property of another where it is reasonably and apparently necessary to avoid threatened injury from a natural or other force and where the threatened injury is substantially more serious than the invasion that seeks to avert it. In cases of private necessity (where the act is solely to benefit a limited number of people rather than the public as a whole) the defense is qualified, so that the actor must pay for any injury she causes. The cyclist was faced with serious injury from being struck by the car door opening. Apparently the only way to avoid this injury was to swerve onto the landowner's yard. The threatened injury to the cyclist was substantially more serious than the cyclist's entry onto the landowner's yard. Thus, the cyclist was privileged to enter the yard. However, because this is a private necessity situation, she will be required to pay for the damage she caused to the lawn ornaments. (A) correctly states that the cyclist is liable for the damage, but incorrectly states that she was not privileged to enter upon the landowner's land. On the other hand, (D) is incorrect because it concludes that the cyclist's privilege absolves her of liability for the damage she caused, which is not true in private necessity cases. (B) is incorrect because the cyclist's exercise of due care is irrelevant. The landowner will be proceeding against the cyclist on a theory of intentional tort (either trespass or conversion). Due care is a concept that is applicable to a negligence action, but is not relevant to an action sounding in intentional tort. Therefore, the cyclist's liability is unaffected by whether she was exercising due care.

A vintner divided his vineyard into two parcels, drawing the boundaries so that the single well that had irrigated the entire vineyard fell on the border of the two properties. The vintner then conveyed the eastern parcel to his friend by a deed that contained the following covenant: "If the well located on the boundary of the eastern and western parcels continues to be used for irrigation purposes and becomes in need of repair or replacement, the grantee, his heirs, and assigns and the grantor, his heirs, and assigns each promise to pay one-half of the cost of such repair or replacement. This covenant shall run with the land." The deed from the vintner to the friend was not recorded, and the vintner did not record a copy of the deed with the records for the western parcel. The friend later sold the eastern parcel to a farmer. The farmer's deed did not contain the covenant about the well. After 15 years of use by the owners of both the eastern and western parcels, the well began to fail. The farmer took it upon himself to have the well repaired at a cost of $30,000. About two weeks later, the farmer discovered the deed from the vintner to the friend in some old files. By this time, the western parcel had passed to the vintner's son by inheritance and again to the son's daughter by inheritance from the now-deceased son. The daughter knew nothing of the covenant concerning the well. The farmer presented the daughter with the bill for the well repair with a copy of the vintner/friend deed and a note that said he expected to be reimbursed for $15,000. The daughter refuses to pay, and the farmer sues. The jurisdiction has a 10-year statute of limitations for acquiring property by adverse possession, and the following recording statute: "Any conveyance of an interest in land shall not be valid against any subsequent purchaser for value, without notice thereof, unless the conveyance is recorded." For whom is the court most likely to rule? A. The daughter, because the deed from the vintner to the friend was never recorded. B. The daughter, because the farmer has acquired the well by adverse possession. C.The farmer, because the covenant runs with the land. D. The farmer, because he is a bona fide purchaser.

C. The farmer, because the covenant runs with the land. The farmer will most likely prevail in his suit for one-half the cost of the well repairs because the covenant runs with the land. When a covenant runs with the land, subsequent owners of the land may enforce or be burdened by the covenant. If all of the requirements for the burden to run are met, the successor in interest to the burdened estate will be bound by the arrangement as effectively as if he had himself expressly agreed to be bound. To be bound: (i) the parties must have intended that the covenant run with the land; (ii) the original parties must have been in horizontal privity; (iii) the succeeding party must be in vertical privity with the original promisor; (iv) the covenant must touch and concern the land; and (v) generally, the burdened party must have actual or constructive notice of the covenant. Here, the intent is shown by the express language of the covenant, which says that it is intended to run with the land. Even without that language, the use of the words "heirs" and "assigns" would show the intent for the covenant to run. The original parties were in horizontal privity because at the time the vintner entered into the covenant, he and the friend shared an interest in the land independent of the covenant—as grantor and grantee. The daughter is in vertical privity with the vintner because she holds the entire interest in the western parcel held by the vintner. The covenant touches and concerns the land because promises to pay money to be used in a way connected with the land are held to touch and concern the property. Because the daughter was unaware of the covenant, the required notice seems to be missing. While it is generally true that the owner of the burdened land must have notice, it should be remembered that the requirement is a function of the recording statute. (At common law, the covenant was enforceable in an action for damages regardless of notice; this was changed by the recording statutes.) However, because the daughter is a donee (an heir) and not a bona fide purchaser, she is not protected by the recording statute and thus is subject to the covenant even without notice.

A driver traveling the speed limit in the evening on a quiet country road rounded a curve and struck a bicyclist who was riding in the same lane. The driver stopped the car and inspected the bicyclist, who had a broken leg. The driver thought it best not to try to move the bicyclist, so he told him that he would go to get help. The driver drove away and left the bicyclist by the side of the road. After the driver had left the scene, he realized that he had forgotten his wife's birthday, so he stopped to buy a gift and hurried home. He did not remember the bicyclist until a few hours later, but assumed that by that time someone would have come along to render assistance. However, the bicyclist was not rescued until the following morning. By then, he had contracted pneumonia as a result of exposure. The bicyclist sued the driver to recover damages for his broken leg and the pneumonia. If the jury finds that the driver was not negligent in his operation of his automobile, for what harm will the bicyclist most likely recover? A. Both the leg injury and the pneumonia. B. The leg injury but not the pneumonia. C. The pneumonia but not the leg injury. D. Neither the leg injury nor the pneumonia.

C. The pneumonia but not the leg injury. The bicyclist will most likely recover for the pneumonia but not for the leg injury. The facts and the call of the question indicate that the driver was not driving negligently when the accident occurred. Therefore, he is not liable for the leg injury caused by the accident, and (A) and (B) are incorrect. However, where the defendant's actions have placed another person in peril or caused another's injury, the defendant has a duty to make reasonable efforts to rescue the imperiled person or render aid to his victim. The driver's neglect of the bicyclist after injuring him will make him liable for the resulting pneumonia.

A buyer and seller entered into a written contract on March 31 for the sale of a beach house. Under the terms of the agreement, the buyer would purchase the house for $275,000, with 10% due at closing on May 1 and a 15-year mortgage. At the time the contract was entered into, the parties agreed orally that the written agreement would not become binding unless the buyer notified the homeowner, in writing, by the end of the day on April 15, that she had secured the proper financing. With the summer season approaching, the seller did not wish to risk any delay in selling the house if the buyer was not in a position to buy it. On the morning of April 15, the buyer's financing was approved. On April 16, the buyer telephoned the seller and told him that her financing had been approved. The buyer also told the seller that she was not able to get written confirmation to him by April 15 because of the postal workers' slowdown and because her fax machine just broke down. The seller assured the buyer that this was not a problem. However, before closing, the seller had a change of heart and decided not to sell the beach house after all. The buyer files an action for breach. Which of the following would be the basis for the buyer's best argument? A. Statute of Frauds. B. Parol evidence rule. C. Waiver of condition. D. Excuse of condition by hindrance.

C. Waiver of condition. The buyer's best argument is that the seller's assurances that there was no problem with the buyer's failure to provide written notification by April 15 amounts to a waiver of the condition. The buyer's written notification by April 15 that he had obtained the proper financing was a condition precedent to the seller's absolute duty to perform under the contract. It is clear that the buyer did not provide the required notification by April 15; thus, the condition was not fulfilled. However, one having the benefit of a condition may indicate by words or conduct that he will not insist upon it. When a condition is broken, the beneficiary of the condition has an election: (i) he may terminate his liability; or (ii) he may continue under the contract. If a choice is made to continue under the contract, the person is deemed to have waived the condition. The seller was fully aware that the buyer had not satisfied the condition, yet, when speaking with the buyer on April 16, he stated unequivocally that it was not a problem. This is a definite indication that the seller elected to continue under the contract. Having so elected, the seller is deemed to have waived the condition. Therefore, the seller's duty of performance under the contract became absolute. Regarding (A), the only way the Statute of Frauds could bolster the buyer's position would be if the original oral agreement setting forth the condition were required by the Statute to be in writing. If that were the case, the buyer could argue that the condition is unenforceable because it is not in writing. However, the oral agreement is not of a type that falls within the purview of the Statute of Frauds. Therefore, the Statute of Frauds will provide no help to the buyer. Similarly, regarding (B), the parol evidence rule could help the buyer only if it could be used to preclude admissibility of the original oral agreement. Under the parol evidence rule, where the parties to a contract express their agreement in a writing with the intent that it embody the final expression of their bargain, any expression made prior to the writing and any oral expression contemporaneous with the writing is inadmissible to vary the terms of the writing. However, where it is asserted that there was an oral agreement that the written contract would not become effective until the occurrence of a condition, evidence of the oral agreement may be offered and received. Because the original oral agreement between the seller and buyer established a condition precedent to the effectiveness of the written agreement, the buyer will be unable to raise the parol evidence rule as a bar to the admissibility of evidence relating to the oral agreement.

While returning from transporting a group of children to summer camp, a bus driver and his assistant were caught in the leading edge of a forest fire raging down the high mountains. Hurrying ahead of the flames and smoke, the driver reached the last half-mile of a dirt road that ran to the main highway and safety, but he discovered that the road ahead was already blocked by fallen, burning foliage. Separating the driver's bus from the main highway, which angled off to the right, was the fenced property of a rancher. The bus driver drove across the property to reach the main highway, damaging some turf and a fence, and proceeded to the city. If the rancher asserts a claim against the bus driver to recover for the damage to his property, is the rancher likely to win? A. No, because the bus driver was acting to protect the lives of himself and his assistant. B. No, because the bus driver acted as would any reasonably prudent person under the circumstances. C. Yes, because the bus driver damaged the rancher's property when he drove through the fence to get to the main highway. D. Yes, because the bus driver intentionally drove across the property, knowing it would cause damage.

C. Yes, because the bus driver damaged the rancher's property when he drove through the fence to get to the main highway. The rancher will probably win. A person who intentionally intrudes upon land in the possession of another is guilty of the tort of trespass to land. The bus driver has clearly met all the elements of this tort. However, the bus driver has the defense of private necessity, because it was necessary to drive onto the rancher's land to avoid the forest fire. Although private necessity is a defense to trespass to lands, it does not relieve the bus driver of liability for damage done to the property.

A hockey player who was playing in the final game of the season before a hostile crowd in the opponent's packed stadium had an opportunity to get his team into the playoffs, but he missed a shot into an open net as the horn sounded, ending the game. As the crowd cheered and jeered, the puck bounced back to him and he shot it in anger toward the stands. A fan who had been looking the other way turned back toward the rink just in time to be struck in the face by the puck. He suffered a broken nose and a severe gash under his eye. After the game, the league commissioner fined the player for violating league rules by intentionally directing the puck out of the playing area. If the fan sues the player for battery, will the fan likely prevail? A. No, because by attending a hockey game, the fan assumed the risk of pucks being shot into the stands. B. No, because the player did not have the intent to strike the fan with the puck. C. Yes, because the player knew that it was substantially certain that a fan would be hit by the puck. D. Yes, because the player violated league rules by intentionally shooting the puck out of the playing area.

C. Yes, because the player knew that it was substantially certain that a fan would be hit by the puck. The fan will prevail in his battery action because the player had the requisite intent for battery. A prima facie case for battery requires plaintiff to prove (i) an act by defendant that brings about a harmful or offensive contact to the plaintiff's person, (ii) intent on defendant's part to bring about harmful or offensive contact, and (iii) causation. The intent element is satisfied as long as the defendant knew with substantial certainty that the harmful or offensive contact would result. Here, the player's conduct caused a harmful contact to the fan, because the player set into motion the force that caused injury to the fan. His intentionally shooting the puck into the crowded stands is enough to establish that he knew with substantial certainty that the puck would strike a spectator. (Note that even if he only intended to cause apprehension of contact, which is the intent for assault, this intent would suffice for liability for battery under the doctrine of transferred intent.) (A) is wrong because assumption of risk is not a defense to intentional torts. The fan may have assumed a risk of injury from a hockey puck's being accidentally or even negligently shot into the stands, but he did not assume the risk of a player's intentionally shooting the puck at a spectator.

The defendant is charged with having been one of two men who robbed a tavern and its patrons at gunpoint at 5:30 p.m. on December 16. The defendant calls a witness to testify that he was at the defendant's house at about 9:30 a.m. on December 16, and that as he was leaving, the defendant said to him, "I'm going to my mother-in-law's house this afternoon for a birthday party." Is the witness's testimony admissible?

Can come in because it is defendant's present state of mind. "Declarations of an existing state of mind are admissible not only when the declarant's state of mind is directly in issue and material to the controversy, but also when the declarant's state of mind is not directly in issue, but the declarations of intent are offered to show subsequent acts of the declarant; i.e., a declaration of intent to do something in the future is offered as circumstantial evidence tending to show that the intent was carried out. The defendant's statement that he was going to his mother-in-law's house is a statement made by the declarant out of court. This statement is offered to prove the truth of the matter asserted therein: that on the day of the alleged armed robbery, the defendant intended to go to his mother-in-law's house. This is being offered as circumstantial evidence that he did go to his mother-in-law's house. Thus, the statement is hearsay. However, the statement does come within the present state of mind exception. Although the defendant's state of mind is not directly in issue, his statement is a declaration of intent to do something offered to show that such intent was in fact carried out."

The plaintiff was driving her daughter to school when their car was struck broadside by a car driven by the defendant at an intersection controlled in all directions by stop signs. The plaintiff and her daughter were taken by ambulance to the hospital. In a personal injury action brought by the plaintiff and her daughter against the defendant, pretrial discovery revealed that both cars were in perfect mechanical condition just before the accident, and the defendant was on his way home from work at the time of the accident, but had stopped off at a bar before he reached the intersection at which he struck the plaintiff's car. There is no witness available to testify as to how much the defendant had to drink at the bar that day. At trial, the plaintiff calls a co-worker of the defendant, who testifies over objection that the defendant has a reputation as a hard drinker who tolerates alcohol well but who always drinks a great deal at any one drinking occasion, as witnessed by the co-worker at numerous company events. Was it error for the trial court to admit his testimony?

Civil case here, as such (unless subject to specific exemptions) cannot introduce character evidence at all. Here, plaintiff attempted to introduce reputation evidence about defendant being a hard drinker. Exceptions: 1. Defamation 2. Civil Fraud 3. Negligent Entrustment 4. Child custody 5. Self-defense for tortious assault/battery

After the release of various news stories about the President's possible violation of political campaign funding laws, a federal grand jury investigation and an investigation by a special Senate subcommittee were initiated.The Senate subcommittee subpoenaed personal records of the President from several top officers of the executive branch. Learning of the subpoenas, the President ordered all executive officials to refuse to turn over the materials. Which of the following statements is most accurate?

Congress can subpoena the President's records but it must advance a legitimate legislative purpose.

A landowner validly conveyed a small office building to the Green Party "as long as they use it for operating quarters until the next presidential election." After the next presidential election, which was in three years, the building would go to a private organization that monitors and prepares comprehensive listings of gas prices throughout the country. A year after the conveyance, the landowner died, validly devising all of her property to her son. Although this jurisdiction is a common law jurisdiction with respect to all real property considerations, the state's probate laws provide that future interests or estates in real property may be passed by will or descent in the same manner as present or possessory interests. Last week, the Green Party and the gas monitoring organization joined together to sell the office building in fee simple absolute to a developer. The son filed suit to prevent the sale of the property to the developer. In this action, who should prevail? A. The Green Party and the gas monitoring organization, because together they own a fee simple absolute in the building. B. The Green Party and the gas monitoring organization, because the attempted restrictions on the use of the property violate the Rule Against Perpetuities. C. The Green Party and the gas monitoring organization, because the deed restriction was an unlawful restraint on alienation. D. The son, because he did not sign the contract of sale.

D. The son, because he did not sign the contract of sale. The son may enjoin the sale because he has an interest in the property. A fee simple determinable is an estate that automatically terminates on the happening of a stated event. The Green Party's interest in the office building is a fee simple determinable because it lasts as long as the Party is using the building for operating quarters. However, the grant does not provide for the contingency of the Green Party ceasing to use the building as operating quarters before the next presidential election. This gap would be filled by a possibility of reverter retained by the landowner. Because the landowner passed that interest to her son in her will, there can be no contract to sell the property without his signature. Note: Although the gas monitoring organization appears to have an indefeasibly vested remainder (i.e., it is created in an ascertained company, is certain to become possessory, and is not subject to being defeated, divested, or diminished in size), its interest is not capable of taking on the natural termination of the preceding estate and so is characterized as a springing executory interest

A plaintiff filed a negligence action against two defendants in federal district court, invoking the court's diversity of citizenship jurisdiction. Prior to serving their answers, the defendants filed a motion to dismiss for lack of subject matter jurisdiction. The court denied the motion, concluding that subject matter jurisdiction was proper. The defendants then filed their answers responding to the merits of the complaint and denying negligence. The defendants' answers also contained motions to dismiss the action for lack of personal jurisdiction. Five weeks later, the defendants filed a motion to dismiss the action for improper venue. If the court determines that venue in fact is improper, how should the court rule on the defendants' venue motion?

Deny the motion! Venue objections are waived if they do not raise such objections in either Rule 12(b) pre-answer motion or the answer itself.

A homeowner offered to pay a roofer $500 to replace the bad shingles on his roof, provided the roofer could finish the job by October 1. The roofer told the homeowner he would get back to him after he had checked out prices at a local supply store. The next day, the roofer phoned the homeowner, who was not at home, and left a message on his answering machine that he could not do the work for less than $650. The roofer did not hear from the homeowner for several days. Because October 1 was still two weeks away, the roofer phoned the homeowner again and left another message on his answering machine stating that he would do the job for $500 and that he would do the work the next weekend unless that would be inconvenient for the homeowner. The homeowner replayed the second message just as he was leaving town on a business trip and did not contact the roofer. That weekend, unbeknownst to the homeowner, the roofer went to the homeowner's house and repaired the roof. When the homeowner returned home, the roofer presented him with a bill for $500, which represented the actual value of the work done. The homeowner refused to pay the bill. If the roofer sues solely for breach of contract, who will likely prevail?

For the homeowner. There was no acceptance! After the counteroffer from the roofer for $650, original offer was terminated. As such, no acceptance to counteroffer means no offer and homeowner prevails. Roofer could maybe prevail on theory of restitution here for unjust enrichment, but not on case "solely" for breach of contract.

A philanthropist told his friend, who was a state governor, that he planned to build a museum. The governor thought that the museum would bolster the state's tourism industry and offered to arrange to have the state purchase land and grant it to the museum to enable the philanthropist to build a bigger museum with his money than originally planned. The philanthropist agreed, and the museum was built. The philanthropist undertook the hiring of the museum's senior staff. He was of German descent and was ashamed of Germany's actions during World War II. To assuage his own conscience, he refused to hire anyone whom he believed to be of German descent. A restoration expert applied for a job as chief curator of the museum, but the philanthropist refused to hire him because of his German background. The restoration expert discovered the philanthropist's rationale and brings suit against the museum, claiming that the hiring practice violates his constitutional rights. How is the court most likely to rule?

For the museum because it is a PRIVATE ENTITY. Apparently acceptance of the funds did not constitute state action or representation, which is what would be required to make this unconstitutional. (C) is incorrect because the grant of land simply does not constitute significant state involvement, which requires that the state affirmatively facilitate, encourage, or authorize the acts of discrimination. Merely granting land to an entity that then decides to adopt a discriminatory policy does not constitute the necessary affirmative action.

A homeowner discovered that the siding on his house was defective and had allowed water to enter the structure, causing damage to the wood framing. The homeowner tried for some time to negotiate a settlement with the corporation that the homeowner believed had manufactured the defective siding. When no settlement was forthcoming, the homeowner filed an action in federal district court against the corporation one week before the statute of limitations expired. Service of process was effected on the corporation several months later. After inspecting the home, the corporation filed and served its answer in which it denied manufacturing the siding used on the homeowner's house. Upon examining the corporation's evidence, the homeowner conceded that the siding was manufactured by another company. With leave of the court, the homeowner then filed an amended complaint substituting the actual manufacturer of the siding for the original incorrect defendant. The amended complaint was served on the manufacturer approximately seven months after the original complaint was filed and after the statute of limitations had expired. The manufacturer was unaware of the action until it was served with the amended complaint. The manufacturer filed a motion for summary judgment on the grounds that the homeowner's claim against it is barred by the statute of limitations. How should the court rule on the motion?

Grant, claim will not relate back in this instance. For a claim to relate back, need: (i) the claims in the amendment arise from the same transaction or occurrence as the claims set out in the original pleading; and (ii) within the time allotted for serving the original complaint (90 days from filing per Federal Rule 4(m)), the new defendant received such notice of the action that it will not be prejudiced in defending on the merits and knew or should have known that the action would have been against it (the new defendant). Here, the manufacturer received no notice of the action within the time allotted for service and could not have known about the plaintiff's mistake in naming the defendant.

A bicyclist was riding his bicycle in the street when a negligently driven car struck the bike, knocking the bicyclist off the bike and breaking his right ankle. The driver of the car immediately stopped and went to his assistance. She got him to his feet and was slowly moving him toward the curb when a negligently driven taxicab struck him in the left leg. The bicyclist required surgery on both his right ankle and his left leg. If the bicyclist sues the driver and the cabbie, which of the following best states his right to recover?

He can recover from either the driver or the cabbie for the injury to his left leg and recover from the driver only for the injury to his right ankle. Because driver's negligence put the cyclist in a position to be injured the second time around, it is a foreseeable intervening cause of injury and he can still be held liable for the second injury. Cabbie, however, cannot be held liable for the first injury because he wasn't even there.

A plumbing contractor sued a homeowner, alleging that the homeowner refused to pay for extensive pipe repairs performed on her home by an employee of the contractor. The contractor called the employee to the stand as a witness. The employee, under oath, testified that he did not perform any work at the homeowner's home. The employee also denied writing a letter to a friend telling the friend that he was going to do plumbing work on the homeowner's house. Without releasing the employee as a witness, the contractor offers in evidence the letter written by the employee to his friend. Which of the following is NOT a proper basis for admitting the employee's letter?

In court comparison by friend, who is a non-expert. Every other way listed is a valid method of having a writing authenticated. "Before a writing may be received into evidence, the writing must be authenticated by proof showing that the writing is what the proponent claims it is. The Federal Rules list several examples of proper methods of authentication through evidence of the genuineness of the handwriting of a letter writer."

An owner of three acres of lakefront property subdivided it and sold two acres to a buyer, retaining the one acre actually fronting on the lake. The deed for the two acres expressly included an easement over the westernmost 30 feet of the one-acre parcel retained by the owner for access to the lake. The buyer recorded his deed in the county recorder's office, which maintained an alphabetical grantor-grantee index only. Fifteen years later, the owner died, leaving the one-acre parcel to his wife. She sold it to a developer that planned to build condominiums. A month later, the buyer died, and his two acres passed by will to his nephew. Three weeks after taking title to the property, the nephew visited the property and discovered that the developer had erected a chain link fence all along the boundary between the nephew's land and the acre of lakefront land. The nephew brings an action to enjoin the developer from obstructing his easement across the acre of lakefront property. Which of the following best describes why the nephew should prevail in this litigation?

Nephew because there is an easement appurtenant here. Where there is an easement appurtenant, it passes with a transfer of the dominant tenement, even though it is an interest in the servient tenement. Thus, the buyer's easement passed to his nephew. Because the easement is perpetual, it is binding on all of the owner's subsequent transferees regardless of whether the conveyance refers to the easement, as long as the transferees have notice of it. Many courts will find record or constructive notice here because the nephew's property is adjacent to the developer's property, is deeded from a common grantor (the owner), and includes the easement in the original deed from the owner.

After a single vehicle accident, the passenger filed a negligence action in federal district court against the driver to recover for a whiplash injury allegedly suffered in the accident. On the advice of his attorney, the passenger consulted and retained five physicians in search of one who would serve as an expert witness on his behalf at trial. Four of the physicians determined that the passenger had suffered no injury. Obviously, the passenger does not intend to use those four physicians as witnesses at trial. May the driver obtain in discovery the opinions of the four physicians whom the passenger does not intend to have testify at trial? Press Enter or Space to submit the answer

No, only under exceptional circumstances can experts obtained in advance of trial not expected to testify be subjected to discovery.

A jogger found a stray dog in the park. She took the dog home with her and placed an ad in the paper to try to find the dog's owner. Soon thereafter, the owner of the dog contacted the jogger. He came to the jogger's home and identified the dog as his. He offered to pay the jogger a $200 reward at the end of the week. The jogger thanked the dog owner but turned down the reward. At the end of the week, however, the jogger changed her mind, so she called the dog owner and told him that she would like the reward after all. He refused to pay her, and she sues him for breach of contract. What will the jogger recover?

PAST CONSIDERATION NO CONSIDERATION AT ALL. Had the owner made an ad, could have been a contract. Here, ad was made by jogger to find the owner. Because he offered to pay after getting the dog, there was no consideration and as such no contract.

Auto workers went on strike in a town heavily reliant on the auto industry. While negotiations between the union and management were ongoing, a person intercepted and recorded a phone call between the union's president and management's chief negotiator. A state statute makes it illegal to record a phone call without the consent of the parties being recorded. The statute also makes it illegal to play an illegally recorded conversation on television or radio. The person who recorded the call anonymously sent the recording to a local TV station. The TV station news anchor played the recording on air. Can the anchor who played the recording be prosecuted under the statute?

Press has constitutional right to public truthful information about matters of public concern! Can only restrict this right by a sanction that is narrowly tailored to further a state interest of the highest order. The right applies even if the information has been unlawfully obtained in the first instance, as long as (1) the speech relates to a matter of public concern, (2) the publisher did not obtain it unlawfully or know who did, and (3) the original speaker's privacy expectations are low.

A landlord entered into a written lease of a bakery for a term of 25 years with a baker. The parties agreed to a right of first refusal if the bakery was offered for sale during the term of the lease. The lease also permitted assignments and subleases on notice to the landlord. Three years later, the baker retired and, after notifying the landlord, transferred the lease to a chocolatier. Twenty-one years later, the landlord entered into a contract with a buyer for the sale of the bakery for $100,000. The landlord had informed the buyer of the lease but had forgotten about the right of first refusal. When the chocolatier learned of the sale to the buyer, she informed both the landlord and the buyer that she wanted to exercise her option and was prepared to purchase the bakery for the contract price. The jurisdiction's Rule Against Perpetuities is unmodified by statute. Can the chocolatier enforce the option?

Right of first refusal was covenant that ran with the land, and because chocolatier was assignee, every covenant in the original contract that ran with the land would be applicable to them as well. Assignee "stands in the shoes of the original tenant"

A homeowner hired a contractor to make some improvements on his house. They entered into a written contract providing that the contractor would do the improvements for $5,000. Shortly after the contract was signed, the contractor told the homeowner to give the money to his (the contractor's) daughter when the job was finished, adding, "She is getting married soon and I want her to have a nice wedding present from me."The daughter was aware that her father made this statement to the homeowner. She married, but soon thereafter the contractor told the homeowner to pay him the $5,000, and not the daughter, because his son-in-law had a gambling problem and would probably use the money to bet at the racetrack. What is the best argument in favor of the daughter's being able to enforce a contract for $5,000 in her favor?

She relied on the promise in getting married, which can be enough to enforce. Note the answer says she will likely lose here

In January, an owner executed and delivered a mortgage on her property to a bank to secure a $50,000 loan. Due to a clerical error, the mortgage was not recorded at that time. On February 15, the owner entered into a contract to sell the property to a buyer for $150,000. On February 16, the owner took out a $30,000 mortgage on the property with a finance company. The finance company promptly and properly recorded its mortgage. Knowing nothing about either of the mortgages, the buyer closed on the property on April 1, tendering $150,000 to the owner. The owner gave the buyer a warranty deed to the property. On April 3, the bank discovered its error and properly recorded its mortgage that same day. The buyer recorded his deed to the property on April 6. The jurisdiction in which the property is located permits mortgages on property under contract, and has a statute that provides: "No conveyance or mortgage of real property shall be valid against a subsequent purchaser for value and without notice whose conveyance is first recorded." The bank brings an appropriate action to determine the status of its mortgage on the property. What should be the court's determination?

The buyer holds the property subject to both mortgages, and the bank's mortgage is subordinate to the finance company's mortgage. Race notice jurisdiction here. Because bank was BFP, and they recorded before buyer, they are attached to property. In pure notice jurisdiction buyer would have only been on hook for finance company because he would have been last BFP for value.

A driver borrowed $75,000 from a bank to purchase a tract of land on which to operate his trucking company, securing the debt with a mortgage on the land. The bank promptly and properly recorded its mortgage. A few years later, the driver financed the installation of a truck wash on the land with a $50,000 loan from a finance company, secured by a mortgage on the land. The finance company promptly and properly recorded its mortgage. The driver subsequently defaulted on the bank's mortgage, leaving an outstanding balance on the bank's loan of $60,000. However, the driver continued to make payments to the finance company. The bank brought a foreclosure action, joining the finance company in the proceeding. The jurisdiction provides a statutory right of redemption for lienholders. Does the finance company have any recourse prior to the foreclosure sale to protect its interest?

The finance company may pay off the bank's mortgage to preserve its own mortgage interest on the land. Because foreclosure will destroy all interests that are junior to the mortgage being foreclosed, the junior mortgagee has the right to pay it off (i.e., redeem it) to avoid being wiped out by its foreclosure. Hence, the finance company may pay off the outstanding balance of the bank's mortgage and be subrogated to the bank's rights against the mortgagor. Statutory right of redemption gives mortgagor and junior lienholders statutory right to redeem for some fixed period after foreclosure sale has occurred.

A man of seemingly modest means died, leaving his nephew as his sole heir. Among the items inherited by the nephew were some old oil paintings. The nephew knew nothing about art and had no place to put the paintings in his home. He placed an ad in the paper offering to sell the paintings at a price to be mutually agreed upon. A buyer for an art gallery responded to the ad. The buyer did not identify himself as an art gallery buyer or tell the nephew that he was knowledgeable about art. Rather, he concocted a story about wanting the paintings for his country estate. The nephew, for his part, revealed his lack of knowledge about art when he told the buyer that his uncle had probably painted the pieces himself. From the signature and the style, the buyer recognized that the artist was a renowned 19th century American portrait artist. The nephew and the buyer agreed upon a price and executed a contract. However, before the nephew delivered the paintings to the buyer, or the buyer paid him, he sought to rescind the contract. The buyer insisted that the nephew deliver the paintings to him and threatened to sue for breach of contract if he did not. Which argument would give the nephew the best basis for rescinding the contract with the buyer?

The nephew revealing he thought his Uncle painted the paintings would put the buyer on notice of a mistake, which could be an excuse to rescind. If buyer has notice of mistake cannot go through with the contract! "The nephew may be able to rescind the contract on the grounds of unilateral mistake if the buyer was aware that the nephew was mistaken about the identity of the artist. Where only one of the parties is mistaken about facts relating to the agreement, the mistake usually will not prevent formation of the contract. However, if the nonmistaken party is aware of the mistake made by the other party, he will not be permitted to snap up the offer; i.e., the mistaken party will have the right to rescind the agreement. Under the facts in this choice, the buyer knows that the nephew is mistaken about the identity of the artist, which is a basic assumption of the contract for the paintings. To obtain rescission, the nephew would also have to establish that the mistake creates a material imbalance in the exchange and that he did not assume the risk of that mistake."

A jogger not paying attention to where he was going collided with an elderly woman in a wheelchair, injuring her. The woman, who was mentally incapacitated, was being pushed down the sidewalk by her daughter, who was also her legal guardian. After the accident, the daughter did not take the woman to the doctor for treatment because the daughter had an unreasonable fear of catching a virus at the doctor's office. As a result, the woman's wounds became infected and took longer to heal. What argument provides the woman with her best chance to recover against the jogger for all of her injuries?

The unreasonable conduct of the daughter is not imputed to the woman! The unreasonable conduct of a Guardian does not prevent recovery for the injured party. Here, even though guardian was unreasonable, Woman should still recover if jogger is found to be liable.

A three-car accident occurred in which the drivers were a citizen of State A, a citizen of State B, and a citizen of State C. The State A citizen filed a negligence action against the other two in federal district court and lost his case. After judgment, may the State C citizen assert and maintain a negligence action against the State B citizen seeking damages for the injuries the State C citizen sustained in the same accident?

Yes because cross-claims are permissive; COUNTER-CLAIMS ARE COMPULSORY

A man was sued by his attorney for monthly retainer fee payments that were past due. The man claimed that he had personally paid the fees every month, in cash, and had receipts to show it. Despite the receipts, the attorney claimed that his records showed no payments from the man on those seven dates. The man subpoenaed the attorney's records of all payments made to the attorney for the days on which the man claimed to have paid his fees, wanting to see if the attorney had entered the payments as being made by another client on those days. The man's subpoena provided that any information concerning the matters handled for other clients be redacted. The attorney moved to quash the subpoena, invoking the attorney-client privilege. Should the court uphold the subpoena?

YES. Information about payments made to attorney by clients is not within scope of the attorney-client privilege. C is incorrect here because the other clients would be protected by the redactions. Only pertinent info was the payment information, not covered by privilege.


Ensembles d'études connexes

Встановлення комуністичного тоталітарного режиму в Україні

View Set

Client Needs: Basic Care and Comfort

View Set

Multiple-choice Questions — Select One Answer Choice

View Set

Skin / Mobility Prep U Leukhardt

View Set

Unit 26-34: 'can'/'could'/'able to'/'should'/'must'/'have'/'need'

View Set

expected range of sugar levels mg/dL?

View Set

Chapter 12: Intra & Postpartum Periods (Test)

View Set

Lecture Chapter 3: Cellular Form and Function

View Set

strangers to these shores chapter 4

View Set